Sei sulla pagina 1di 66

Polinomi ortogonali

Dario A. Bini Paola Boito Beatrice Meini


17 marzo 2021

1 Definizione e proprietà
Nel seguito si denota con Pn lo spazio vettoriale di dimensione n + 1 costituito
dai polinomi a coefficienti reali di grado al più n e con P lo spazio vettoriale di
tutti i polinomi a coefficienti reali.

Definizione 1 Dato un prodotto scalare h·, ·i, su P, un insieme di polinomi


{pi (x) ∈ Pi , i = 0, 1 . . .} tale che deg(pi ) = i e hpi , pj i = 0 se i 6= j è detto
insieme di polinomi ortogonali relativamente al prodotto scalare h·, ·i. I polinomi
1/2
pi (x)/hi , hi = hpi , pi i sono detti polinomi ortonormali. 

Il prodotto scalare h·, ·i induce la norma su P

kpk = hp, pi1/2 , p ∈ P.

Se associamo a p(x) ∈ Pn la (n + 1)-upla dei suoi coefficienti, possiamo


rappresentare lo spazio lineare Pn con Rn+1 . Per cui ogni prodotto scalare su
Rn+1 induce un prodotto scalare su Pn . In particolare il prodotto scalare
Pn euclideo
su Rn+1Pinduce il prodotto scalare tra polinomi hp(x), q(x)i = i=0 p i qi dove
n Pn
p(x) = i=0 pi xi , q(x) = i=0 qi xi . In particolare, i polinomi xi , i = 0, 1, . . .,
sono ortonormali rispetto al prodotto scalare indotto dal prodotto euclideo.

Osservazione 2 Dato un generico prodotto scalare su P, esistono sempre poli-


nomi ortogonali che possono essere costruiti mediante il procedimento di ortogo-
nalizzazione di Gram-Schmidt a partire dai monomi 1, x, x2 , x3 , . . ., nel seguente
modo:
Pk−1
p0 (x) = 1, pk (x) = xk + i=0 αk,i pi (x),

k = 1, 2, . . . (1)
αk,i = − xk , pi (x) / hpi (x), pi (x)i , i = 0, . . . , k − 1.

Vedremo successivamente, introducendo altri prodotti scalari, che ci sono tecniche


più efficienti per la costruzione e la manipolazione di polinomi ortogonali. 

1
La seguente banale osservazione ha una serie di conseguenze meno immediate
e di particolare rilevanza.
Osservazione 3 I polinomi ortogonali p0 (x), p1 (x), . . . , pn (x) sono linearmente
indipendenti e quindi costituiscono una base dello spazio vettoriale Pn dei poli-
nomi di grado minore o uguale ad n. 

Dalla precedente osservazione segue il


Teorema 4 Se p0 , p1 , . . . , sono polinomi ortogonali allora per ogni polinomio
q ∈ Pn vale hpi , qi = 0 per ogni i > n.
Pn
Dim. Per l’osservazione 3 si ha q(x) = j=0 γj pj (x). Se i > n vale allora
Pn
hpi , qi = j=0 γj hpi , pj i = 0. 

Vale il seguente risultato di minima norma


Teorema 5 Tra tutti i polinomi p(x) di grado n che hanno coefficiente del
termine di grado massimo uguale a quello di pn (x), il polinomio pn (x) è quello
di minima norma.
Dim. Ogni polinomio q di grado n che ha lo stesso coefficiente di grado massimo
di pn (x) si lascia scrivere come q(x) = pn (x) + qn−1 (x), dove qn−1 (x) è un
polinomio di grado al più n − 1. Vale quindi
kqk2 = hq, qi = hpn , pn i + hqn−1 , qn−1 i
essendo hqn−1 , pn i = 0 per l’ortogonalità. Risulta allora kqk2 ≥ kpn k2 dove
l’uguaglianza è raggiunta se e solo se qn−1 = 0 cioè q = pn . 

Vogliamo ora definire un diverso prodotto scalare, che sarà utile nello sviluppo
della teoria dell’approssimazione di funzioni.
Siano a, b due costanti della retta reale estesa R = R ∪ {+∞, −∞} con a < b.
In questo modo l’intervallo [a, b] può essere anche del tipo [−∞, β], [α, +∞], con
α, β ∈ R, e [−∞, +∞]. Sia ω(x) : [a, b] → R una funzione a valori nella retta
reale estesa tale che per a < x < b valga ω(x) ∈ R, ω(x) > 0 ed esista finito
Rb
a
f (x)ω(x)dx per ogni polinomio f (x) ∈ P. La funzione ω(x) è detta funzione
peso.
L’applicazione che a una coppia di polinomi (f (x), g(x)), f (x), g(x) ∈ P
associa il numero reale
Z b
hf, gi := f (x)g(x)ω(x)dx (2)
a

è un prodotto scalare sul P. Inoltre l’applicazione che a f (x) ∈ P associa


!1/2
Z b
f (x)2 ω(x)dx
a

2
è una norma su P che viene denotata con kf (x)k.
Esempi classici di funzioni peso che definiscono
√ il prodotto scalare√di tipo
integrale sono ω(x) = 1 su [−1, 1], ω(x) = 1 − x2 su [−1, 1], ω(x) = 1/ 1 − x2
2
su [−1, 1], ω(x) = e−x su [0, +∞], e−x su [−∞, +∞].

Osservazione 6 Il prodotto scalare appena introdotto verifica la proprietà


hxf (x), g(x)i = hf (x), xg(x)i. Non tutti i prodotti scalari su P verificano questa
proprietà. Ad esempio, il prodotto scalare indotto dal prodotto scalare eucli-
deo sui coefficienti non soddisfa la proprietà hxf (x), g(x)i = hf (x), xg(x)i per
f (x), g(x) ∈ Pn−1 . 

Osservazione 7 Col prodotto scalare integrale (2) per calcolare i coefficienti


αk,i per i = 0, . . . , k − 1 e k = 1, . . . , n nell’Osservazione 2 occorre calcolare,
o approssimare numericamente, n(n + 1)/2 integrali. Inoltre, per calcolare
i coefficienti del generico polinomio pk di grado k occorre eseguire circa k 2
operazioni aritmetiche per cui il costo totale per calcolare i coefficienti di tutti
i polinomi pk , k = 0, . . . , n è di O(n3 ) operazioni. Questo però non è il modo
migliore di procedere. Come vedremo tra poco ci sono espressioni più semplici
per esprimere i coefficienti di pk (x).
Inoltre per calcolare il valore che pk (x) assume in un punto ξ usando la
(1) occorrono circa 2k operazioni aritmetiche avendo però prima calcolato i
valori di pi (ξ), i = 0, 1, . . . , k − 1 e i coefficienti ak,i . Quindi occorrono O(k 2 )
operazioni. Anche questo calcolo può essere semplificato mediante una diversa
rappresentazione dei polinomi ortogonali. 

La proprietà espressa nel Teorema 4 ci permette di dimostrare il seguente


risultato interessante.

Teorema 8 Gli zeri dei polinomi ortogonali rispetto al prodotto scalare (2), di
grado almeno 1, sono reali e semplici e stanno in (a, b).

Dim. Sia n ≥ 1 e siano x1 , . . . , xj , 0 ≤ j ≤ n, gli zeri reali e distinti di pn (x)


in (a, b), con la convenzione che, se j = 0, pn (x) non ha zeri reali in (a, b) . Per
assurdo supponiamo j < n. Supponiamo inoltre che k di questi zeri, 0 ≤ k ≤ j,
abbiano molteplicità dispari e j − k pari. Numeriamo gli xi in modo che i primi
k abbiano molteplicità dispari. Definiamo
(
1 se k = 0
q(x) = Qk
s=1 (x − xs ) se k > 0.

Allora gli zeri reali in (a, b) di pn (x)q(x) hanno molteplicità pari e quindi questo
prodotto non cambia segno in (a, b). Conseguentemente
Z b
hpn , qi = pn (x)q(x)ω(x)dx 6= 0
a

3
che è assurdo per il teorema 4 poiché il grado di q(x) è k ≤ j < n. 

Un’altra conseguenza del teorema 4 è data dal seguente risultato che ha


rilevanza sia teorica che computazionale.
Teorema 9 (Ricorrenza a tre termini) Siano pi (x), i = 0, 1, . . ., un insie-
me di polinomi ortogonali su [a, b] rispetto a un prodotto scalare (2). Si denoti
p0 (x) = a0 , p1 (x) = a1 x + b1 . Esistono Ai , Bi , Ci ∈ R tali che
pi+1 (x) = (xAi+1 + Bi+1 )pi (x) − Ci pi−1 (x), i ≥ 1, Ai+1 , Ci 6= 0.
Inoltre
hpi+1 , pi+1 i hxpi , pi i
Ai+1 = , Bi+1 = −Ai+1 ,
hxpi , pi+1 i hpi , pi i
hxpi , pi−1 i Ai+1 hpi , pi i
Ci = Ai+1 = .
hpi−1 , pi−1 i Ai hpi−1 , pi−1 i
Denotando con ai e bi rispettivamente i coefficienti di xi e xi−1 in pi (x), vale
Ai+1 = aai+1 i
, Bi+1 = aai+1
i
( abi+1
i+1
− abii ), Ci = ai+1aa2 i−1 hhi−1
i
= AAi+1 hi
i hi−1
, dove
i
hi = hpi , pi i.
Dim. Il polinomio xpi (x) ha grado i + 1 per cui xpi , pi , . . . , p0 sono linearmente
indipendenti. Esistono allora α0 , α1 , . . . , αi+1 ∈ R tali che
pi+1 = α0 p0 + · · · + αi pi + αi+1 xpi .
Per 0 ≤ j ≤ i vale
0 = hpj , pi+1 i = αj hpj , pj i + αi+1 hpj , xpi i = αj hpj , pj i + αi+1 hxpj , pi i .
Poiché se j ≤ i − 2 il polinomio xpj ha grado minore di i, per il teorema 4 è
hxpj , pi i = 0. Quindi αj = 0 per j = 0, . . . , i − 2 per cui
pi+1 = αi+1 xpi + αi pi + αi−1 pi−1 = (αi+1 x + αi )pi + αi−1 pi−1 . (3)
Vale quindi il risultato del teorema con Ai+1 = αi+1 , Bi+1 = αi Ci = −αi−1 .
Le costanti Ai , Bi , Ci possono essere facilmente espresse in termini di prodotti
scalari. Infatti moltiplicando scalarmente per pi+1 , pi e pi−1 entrambi i membri
della (3) si ottiene
hpi+1 , pi+1 i = Ai+1 hxpi , pi+1 i,
Ai+1 hxpi , pi i + Bi+1 hpi , pi i = 0
Ai+1 hpi−1 , xpi i − Ci hpi−1 , pi−1 i = 0
da cui hxpi (x), pi+1 (x)i 6= 0 e si ricavano le espressioni per Ai+1 , Bi+1 , Ci . Le
altre relazioni seguono da un confronto diretto dei polinomi. Inoltre, poiché
hpi ,pi i
6 0 ne segue che Ai+1 6= 0, inoltre dall’espressione Ci = AAi+1
hpi+1 , pi+1 i = i hpi−1, ,pi−1 i
segue Ci 6= 0. 

4
Osservazione 10 Poiché i polinomi del teorema 9 sono definiti a meno di una
costante moltiplicativa, possiamo scegliere i polinomi monici in modo che ai = 1
per ogni i ≥ 1 per cui Ai = 1 per i ≥ 1. In questo modo risulta

hxpi , pi i hpi , pi i
Ai+1 = 1, Bi+1 = − , Ci = > 0.
hpi , pi i hpi−1 , pi−1 i

Osservazione 11 Si noti che la relazione a tre termini data nel teorema 9 ci


dice che pi−1 (x) è il resto della divisione di pi+1 (x) per pi (x). Per cui i polinomi
ortogonali pn+1 (x), pn (x), . . . , p0 (x) possono essere visti come i polinomi generati
dall’algoritmo Euclideo applicato a pn+1 (x) e pn (x). 

Osservazione 12 Fissato i, i coefficienti Ai+1 = 1, Bi+1 , Ci richiedono il


calcolo dei prodotti scalari hpi , pi i, hpi−1 , pi−1 i, hxpi , pi i. Per cui il loro calcolo
per i = 1, . . . , k comporta solamente 2k prodotti scalari.
Per calcolare il valore di pk (ξ) usando la relazione a tre termini, supponendo
di avere a disposizione i coefficienti Ai+1 = 1, Bi+1 e Ci e di aver calcolato i
valori pi (ξ) per i = 0, . . . , k − 1, bastano 4 operazioni aritmetiche. Quindi per
calcolare tutti i valori pi (ξ) per i = 1, . . . , k bastano 4k operazioni aritmetiche.
In modo analogo si vede che i coefficienti di tutti i polinomi ortogonali di
grado al più k sono calcolabili in O(k 2 ) operazioni.
Un confronto col metodo di ortogonalizzazione di Gram-Schmidt mostra
che l’uso della relazione ricorrente a tre termini abbassa sostanzialmente i costi
computazionali delle operazioni con polinomi ortogonali. 

Teorema 13 (Formula di Christoffel-Darboux) Siano pi (x), i = 0, 1, . . .,


un insieme di polinomi ortogonali su [a, b] rispetto a un prodotto scalare (2).
Allora, per ogni n ≥ 0, vale
n
X 1
(x − y) pi (x)pi (y) = γn [pn+1 (x)pn (y) − pn+1 (y)pn (x)] (4)
h
i=0 i

an 1
dove γn = an+1 hn = hn An+1 , hi = hpi , pi i. Se normalizziamo i polinomi in modo
che siano monici, e quindi scegliamo a0 = a1 = 1, √ Ai = 1, i ≥ 1, si ottiene
γn = 1/hn . Se inoltre denotiamo con pbn (x) = pn (x)/ hn il generico polinomio
normalizzato in modo che hbpn (x), pbn (x)i = 1, la formula di Christoffel-Darboux
diventa
n
X an
(x − y) pn (y) − pbn+1 (y)b
b
pbi (x)b
pi (y) = (b
pn+1 (x)b pn (x)),
i=0
an+1
b

dove b
ai indica il coefficiente principale di pbi (x).

5
Dim. Procediamo per induzione. Se n = 0 la relazione diventa
a20
(x − y) = γ0 ((a1 x + b1 )a0 − (a1 y + b1 )a0 )
h0
da cui γ0 = aa1 h0 0 che è verificata per ipotesi.
Per l’implicazione n − 1 → n, consideriamo l’espressione pn+1 (x)pn (y) −
pn+1 (y)pn (x) al secondo membro di (4), e sostituiamo al posto di pn+1 (x) e di
pn+1 (y) l’espressione ottenuta con la relazione a tre termini. Si ottiene quindi
pn+1 (x)pn (y) − pn+1 (y)pn (x) = ((xAn+1 + Bn+1 )pn (x) − Cn pn−1 (x))pn (y)
− ((yAn+1 + Bn+1 )pn (y) − Cn pn−1 (y))pn (x).
Semplificando si arriva a
(x − y)An+1 pn (x)pn (y) + Cn (pn (x)pn−1 (y) − pn (y)pn−1 (x))
Per l’ipotesi induttiva il secondo addendo coincide con
n−1
Cn X 1
(x − y) pi (x)pi (y)
γn−1 h
i=0 i

per cui possiamo scrivere


pn+1 (x)pn (y) − pn+1 (y)pn (x) =
n−1
!
1 Cn X 1
(x − y) An+1 hn pn (x)pn (y) + pi (x)pi (y) .
hn γn−1 i=0 hi
Cn
Basta quindi verificare che An+1 hn + γn−1 = γn . Questo segue dal fatto che
An+1 hn
Cn = An hn−1 valida per il Teorema 9 e dall’espressione di γn−1 .


Osservazione 14 Una dimostrazione alternativa della formula di Christoffel-


Darboux è la seguente. Vale
pi+1 (x) = (Ai+1 x + Bi+1 )pi (x) − Ci pi−1 (x) moltiplico per pi (y)
pi+1 (x)pi (y) = (Ai+1 x + Bi+1 )pi (x)pi (y) − Ci pi−1 (x)pi (y) scambio variabili
pi+1 (y)pi (x) = (Ai+1 y + Bi+1 )pi (y)pi (x) − Ci pi−1 (y)pi (x) sottraggo
pi+1 (x)pi (y) − pi+1 (y)pi (x)
= pi (x)pi (y)[x − y]Ai+1 − Ci [pi−1 (x)pi (y) − pi−1 (y)pi (x)]
Quindi
[x − y]pi (x)pi (y)Ai+1 = pi+1 (x)pi (y) − pi+1 (y)pi (x)
+ Ci [pi−1 (x)pi (y) − pi−1 (y)pi (x)]
1 1
[x − y]pi (x)pi (y) = [pi+1 (x)pi (y) − pi+1 (y)pi (x)]
hi hi Ai+1
Ci
+ [pi−1 (x)pi (y) − pi−1 (y)pi (x)]
hi Ai+1

6
dove Ci = (hi /hi−1 )(Ai+1 /Ai ). Si ottiene allora
1 1
[x − y]pi (x)pi (y) = [pi+1 (x)pi (y) − pi+1 (y)pi (x)]
hi hi Ai+1
1
+ [pi−1 (x)pi (y) − pi−1 (y)pi (x)]
hi−1 Ai

Sommando, i termini si elidono a due a due e si ottiene la tesi. 

Osservazione 15 Applicando la formula di Christoffel Darboux per x = xi e


y = xj , dove x1 , . . . , xn+1 sono gli zeri di pn+1 (x), si ottiene
n n
X 1 X
pk (xi )pk (xj ) = pk (xj ) = 0, i 6= j,
pbk (xi )b
hk
k=0 k=0
√ Pn
dove pbi (x) = p(x)/ hi . Inoltre k=0 pk (xi )2 /hk = 6 0 essendo p0 (xi )2 /h0 =
b
a20 /h0 = 1/ a ω(x)dx > 0. La matrice V = (vi,j )i,j=1,n+1 , vi,j = pbi−1 (xj ) è tale
R

che V T V è una matrice diagonale. Cioè V ha le colonne v (j) = (vi,j )i ortogonali.


Possiamo scalare le colonne in modo da farle diventare ortonormali, basta per
questo dividerle per la loro norma euclidea. Posto
n
X
σj = 1/kv (j) k, kv (j) k2 = pbi (xj )2
i=0

la matrice W = V diag(σ1 , . . . , σn+1 ) ha colonne ortonormali e verifica la pro-


prietà W T W = I per P cui è ortogonale e quindi W W T = I. Quest’ultima
n+1
relazione si scrive come k=1 wi,k wj,k = δi,j , da cui
n+1 n
X X 1
pj (xk )σk2 = δi,j ,
pbi (xk )b σk2 = 1/ ps (xk )2
h s
k=1 s=0 (5)
1
pbi (x) = √ pi (x), hi = hpi (x), pi (x)i.
hi
Questa espressione fornisce una sorta di ortonormalità “discreta” dei polinomi
pbi (x) relativa ai punti xk e ai pesi σk2 . In particolare, se associamo ai polinomi
(j)
pbj (x) i vettori u(j) = (ui )i per j = 0, . . . , n, ottenuti campionando i polinomi
(j)
ortonormali negli zeri di pn+1 (x), cioè tali che ui = pbj (xi ), abbiamo che
(0) (n)
u ,...,u sono ortonormali rispetto al prodotto scalare (discreto pesato)
n+1
X
hy, zi = yi zi σi2 .
i=1

7
Osservazione 16 Dati due polinomi u(x) e v(y) il bezoutiano B(x, y) di u(x) e
v(x) è il polinomio in due variabili (u(x)v(y)−u(y)v(x))/(x−y). Il bezoutiano ha
diverse particolarità interessanti che sono utili in computer algebra. La formula
di Cristoffel-Darboux aggiunge una ulteriore proprietà: se u e v sono polinomi
ortogonali di grado n + 1 e n il loro bezoutiano è somma di prodotti di polinomi
ortogonali. Si associa al bezoutiano B(x, y) la matrice Bn = (bi,j ) tale che
Pn+1 Pn+1
B(x, y) = i=1 j=1 bi,j xi−1 y j−1 chiamata matrice bezoutiana. È immediato
verificare che Bn è simmetrica e che B(x, y) = xT Bn y dove x = (xi )i=0,n−1 ,
y = (y i )i=0,n−1 . Poiché pk (x)pk (y) = xT p(k) p(k)T y, dove p(k) è il vettore dei
coefficienti del polinomio pk (x), la formula di Christoffel Darboux permette di
scrivere la matrice bezoutiana Bn di pn (x) e pn+1 (x) come somma delle n diadi
simmetriche γn1hk p(k) p(k)T . La stessa somma di diadi simmetriche permette
di scrivere la matrice bezoutiana come γn Bn = LT L, dove L è una matrice
triangolare inferiore√che sulla riga i-esima ha i coefficienti dell’i-esimo polinomio
ortonormale pi (x)/ hi per i = 0, . . . , n.
Una proprietà interessante della matrice bezoutiana di due polinomi qualsiasi
u(x) e v(x)è che la sua fattorizzazione UL a blocchi fornisce quozienti e resti
generati dall’algoritmo euclideo applicato a u(x) e v(x). Inoltre la matrice
bezoutiana è non singolare se e solo se i polinomi u(x) e v(x) sono primi tra loro.
Maggiori informazioni sul bezoutiano, per chi volesse approfondire, si trovano
sui lavori:
H.K. Wimmer, “On the History of the Bezoutian and the Resultant Matrix”,
Linear Algebra Appl. 128:27-34(1990),
U. Helmke, P.A. Fuhrmann, “Bezoutians”, Linear Algebra Appl. 122–124:1039-
1097(1989),
G. Heinig, K. Rost, Bezoutians, https://www.tu-chemnitz.de/mathematik/
preprint/.../PREPRINT_09.pdf
Onorato Nicoletti, “Sulla caratteristica delle matrici di Sylvester e di Be-
zout”, Da una lettera al Prof. Alfredo Capelli, 27 Dicembre 1908 http:
//link.springer.com/article/10.1007%2FBF03018209#page-1 

2 Polinomi ortogonali e matrici tridiagonali


Sia Tn (x) la matrice tridiagonale n × n definita da
 
a1 x + b1 −a0
 −C1 A 2 + B2 −1
x 
 
 .. .. .. 
Tn (x) = 
 . . . ,

 . .. . ..

 −1 
−Cn−1 An x + B n

dove Ai , Bi e Ci sono i coefficienti che intervengono nella ricorrenza a tre termini.


Applicando la regola di Laplace per calcolare det Tn (x) lungo l’ultima riga si

8
scopre che

det Tn (x) = (An x + Bn ) det Tn−1 (x) − Cn−1 det Tn−2 (x)

inoltre det T1 (x) = a1 x + b1 = p1 (x) e

det T2 (x) = (A2 x + B2 )p1 (x) − C1 p0 (x) = p2 (x).

Per cui, ragionando per induzione, vale det Tn (x) = pn (x).


È interessante osservare che
   
p0 (x) 0
 p1 (x)   .. 
Tn (x) 

.. =
  . 

 .   0 
pn−1 (x) pn (x)

per cui il vettore di componenti (pi (x))i=0:n−1 sta nel nucleo di Tn (x) se e solo
se x è uno zero di pn (x).

Osservazione 17 Se i polinomi ortogonali vengono normalizzati in modo che


Ai = 1 per ogni i, allora Tn (x) = xI −Tn , dove Tn := tridn (Ci , −Bi , 1), e dove per
semplicità di notazione si è posto B1 = b1 . Dunque gli zeri di pn (x) possono essere
visti come gli autovalori della matrice Tn e gli autovettori u(i) corrispondenti,
tali che Tn u(i) = xi u(i) , hanno per componenti i valori dei polinomi ortogonali
calcolati negli zeri di pn (x), cioè u(i) = (p0 (xi ), p1 (xi ), . . . , pn−1 (xi ))T .
Si osservi ancora che la matrice ottenuta cambiando segno agli elementi sopra
e sotto diagonali alla matrice Tn ha gli stessi autovalori della matrice originale e
autovettori con componenti a segno alterno. 

Osservazione 18 Assumendo di aver normalizzato i polinomi in modo che


a0 = Ai = 1, i = 1, . . . , n, poiché i Ci sono tutti positivi per l’osservazione 10,
è possibile determinare una matrice diagonale D = diag(d1 , . . . , dn ) tale che
D−1 Tn D è simmetrica. Infatti, imponendo la condizione di simmetria, otteniamo √
le condizioni Ci di /di+1 = di+1 /di , i = 1, . . . , n − 1, da cui di+1 =√ di Ci ,
i = 1, . . . , n − 1. Nell’ipotesi
p di monicità
p dei polinomi,
p scegliendo
√ d1 = h0 = 1,
otteniamo di+1 = hi /hi−1 di = hi /hi−1 hi−1 = hi , dove hi = hpi , pi i,
i = 0, . . . , n − 1. Inoltre vale

b1√+ x − C1
 

 − C1 B2 + x − C2 
 
.. .. ..
D−1 Tn (x)D = 
 
 . . . .

 . .. . .. p 

p − Cn−1 
− Cn−1 Bn + x

Gli autovettori v (j) tali che D−1 Tn (xj )Dv (j) = 0 sono v (j) = D−1 u(j) con u(j) =
(p0 (xj ), . . . , pn−1 (xj ))T . Cioè l’i-esimo autovettore della matrice simmetrizzata

9
D−1 Tn D, corrispondente all’autovalore xi , è

v (i) = (b
p0 (xi ), pb1 (xi ), . . . , pbn−1 (xi ))T ,

dove pbi (x) = √1h pi (x) sono i polinomi ortonormali.


i
Poiché gli autovettori di una matrice reale simmetrica con autovalori distinti
sono ortogonali, ne segue l’ortogonalità dei vettori v (j) e quindi
n
X 1
pk (xi )pk (xj ) = 0, i 6= j.
hk
k=0

Questa proprietà era stata ottenuta anche dalla formula di Christoffel Darboux. 

Le proprietà degli zeri dei polinomi ortogonali sono quindi strettamente


legate alle proprietà degli autovalori delle matrici tridiagonali simmetriche. Nel
prossimo paragrafo riportiamo un teorema molto utile, di interesse generale, che
ci permette di scoprire proprietà interessanti degli zeri dei polinomi ortogonali

2.1 Il teorema del minimax di Courant-Fischer


Data A ∈ Rn×n simmetrica e x ∈ Rn , x 6= 0 si definisce il quoziente di Rayleigh
di A in x l’espressione
xT Ax
r(x) = T .
x x
Il quoziente di Rayleigh può essere visto come un’applicazione Rn \ {0} → R.

Osservazione 19 Il gradiente del quoziente di Rayleigh è


2
(A − λI)x, λ = r(x).
xT x
Infatti, poiché A è simmetrica, vale
∂ T ∂
∂r(x) ∂xj (x Ax) (xT Ax) ∂x j
(xT x)
= −
∂xj xT x (xT x)2
(Ax)j (xT Ax)xj 2
=2 T
−2 = T (Ax − r(x)x)j .
x x (xT x)2 x x

Quindi i punti stazionari del quoziente di Rayleigh sono gli autovettori di


A e i valori presi dal quoziente in corrispondenza dei punti stazionari sono i
corrispondenti autovalori. Questo porta al seguente primo risultato

10
Teorema 20 Siano λ1 ≥ · · · ≥ λn gli autovalori della matrice simmetrica
A ∈ Rn×n . Vale
xT Ax xT Ax
max T = λ1 , min T = λn .
x6=0 x x x6=0 x x
Si può dimostrare molto di più:

Teorema 21 (Teorema di Courant-Fischer del minimax) Siano λ1 ≥ · · · ≥


λn gli autovalori della matrice simmetrica A ∈ Rn×n . Indichiamo con Vk un
generico sottospazio vettoriale di Rn di dimensione k. Vale
xT Ax
min max = λn−k+1 , k = 1, 2, . . . , n
Vk ⊂ Rn x ∈ Vk xT x
dim Vk = k x 6= 0

xT Ax
max min = λk , k = 1, 2, . . . , n.
Vk ⊂ Rn x ∈ Vk xT x
dim Vk = k x 6= 0

Dim. Si dimostra la versione maxmin. Siano xi gli autovettori di A ortonormali


e S=span(xk , . . . , xn ). Allora per ogni sottospazio Vk di dimensione k vale
S ∩ VP
k 6= {0} poiché dim(S) = n − k + 1 e dim(Vk ) = k. Allora esiste x ∈ S ∩ Vk ,
n
x = i=k αi xi 6= 0. Vale
Pn 2
Pn
xT Ax i=k αi λi αi2
T
= P n 2 ≤ λk Pi=k
n 2 = λk .
x x i=k αi i=k αi

Quindi il minimo su ogni Vk del quoziente di Rayleigh è minore o uguale a λk


e quindi anche il massimo al variare di Vk dei minimi è minore o uguale a λk .
Basta quindi fare vedere che esiste un Vbk speciale in cui il minimo del quoziente
di Rayleigh vale λk . Per questo basta scegliere Vbk =span(x1 , . . . , xk ). Infatti
Pk
per x ∈ Vbk risulta x = i=1 αi xi per cui
Pk 2 T
xT Ax i=1 λi αi xi xi
= k
≥ λk .
xT x 2 T
P
i=1 αi xi xi

La versione minmax del teorema si ottiene applicando la versione maxmin alla


matrice −A. 

Alcuni corollari interessanti del teorema del minimax.

Corollario 22 Sia A una matrice reale simmetrica n × n di autovalori α1 ≥


· · · ≥ αn . Sia U ∈ Rn×(n−1) tale che U T U = In−1 . Allora gli autovalori
β1 ≥ · · · ≥ βn−1 di B = U T AU sono tali che

α1 ≥ β1 ≥ α2 ≥ β2 ≥ · · · ≥ βn−1 ≥ αn .

e si dice che gli autovalori di B separano gli autovalori di A.

11
Dim. Indichiamo con Wk un generico sottospazione di Rn−1 di dimensione k.
Vale
y T By y T By y T U T AU y
βk = max min = min = min
Wk y ∈Wk \{0} y T y ck \{0} y T y
y ∈W y ∈W
ck \{0} yT y

dove W
ck è il sottospazio dove viene preso il massimo. Sia

Vbk = {x ∈ Rn : x = U y, y ∈ W
ck }.

Vale dim(Vbk ) = k e
y T U T AU y xT Ax
=
yT y xT x
per cui
y T U T AU y xT Ax xT Ax
βk = min = min ≤ max min = αk .
y ∈W
ck \0 yT y x∈Vbk \0 xT x Vk x∈Vk \0 xT x

La diseguaglianza βk−1 ≥ αk si ottiene applicando il risultato a −A e −B. 

Corollario 23 Se A è una matrice tridiagonale simmetrica n × n, allora gli


autovalori di una qualsiasi sottomatrice principale di A di dimensione (n − 1) ×
(n − 1) separano gli autovalori di A.
Come conseguenza del teorema del minimax segue l’importante proprietà
degli zeri dei polinomi ortogonali
Teorema 24 Gli zeri del polinomio ortogonale pn (x) separano strettamente gli
zeri del polinomio ortogonale pn+1 (x), per ogni n ≥ 1.
Dim. La separazione con diseguaglianza debole segue dal corollario 23 appli-
cato alla matrice tridiagonale simmetrica ottenuta simmetrizzando la matrice
tridn (−Ci , Bi , −1) alla luce delle osservazioni 17, 18. Per dimostrare la disegua-
glianza stretta si supponga per assurdo che λ sia zero di pn+1 e di pn . Dalla
relazione a tre termini segue che λ è zero di pn−1 . Procedendo per induzione si
conclude che λ è zero di p0 che è assurdo. 

Si riportano a titolo di curiosità due altri corollari di cui non si dà dimostra-
zione.
Corollario 25 Se A, B, C sono matrici reali simmetriche tali che A = B + C
allora per gli autovalori αi , βi , γi ordinati in modo non crescente vale
βi + γn ≤ αi ≤ βi + γ1 .
Corollario 26 Se A, B, C sono matrici reali simmetriche tali che A = B + C e
C = uuT per un vettore nonnullo u, allora per gli autovalori αi , βi di A e B
ordinati in modo non crescente vale
βi ≤ αi ≤ βi−1 , i = 1, . . . , n,
dove β0 = β1 + uT u.

12
3 Rappresentazione di polinomi ortogonali
Oltre alla rappresentazione data mediante la relazione ricorrente a tre termini
esistono altri modi per rappresentare i polinomi ortogonali. In questo paragrafo
esaminiamo due rappresentazioni diverse, quella basata sulla matrice dei momenti
e quella data dalla formula di Rodrigues.

3.1 Matrice dei momenti


Rb
Le quantità µk = a
xk ω(x)dx, k = 0, 1, . . . , n, vengono dette momenti. Vale il
seguente risultato
Teorema 27 I polinomi pn (x) = det Mn (x), dove
 
µ0 µ1 µ2 ... µn
 µ1 µ 2 µ 3 ... µn+1 
 
 µ2 µ 3 µ 4 ... µn+2 
Mn (x) =  .  , n = 0, 1, . . . (6)
 
. . ..
 .. .. .. ... . 
 
 µn−1 µn µn+1 . . . µ2n−1 
1 x x2 ... xn
Rb
sono ortogonali rispetto al prodotto scalare hf, gi = a
f (x)g(x)ω(x)dx.

Dim. Basta verificare che hxk , pn (x)i = 0 per k = 0, 1, . . . , n − 1. Si osserva che


Rb Rb 
hxk , pn (x)i = a xk ω(x) det Mn (x)dx = a det diag(1, . . . , 1, xk ω(x))Mn (x) dx.
La matrice diag(1, . . . , 1, xk ω(x))Mn (x) ha le prime n righe costanti e l’ultima
Rb 
riga dipendente da x, dunque a det diag(1, . . . , 1, xk ω(x))Mn (x) dx coincide
con il determinante della matriceR che si ottiene sostituendo l’ultima riga del-
b k Rb Rb
la matrice Mn (x) in (6) con a x ω(x)dx, a xk+1 ω(x)dx, . . . , a xk+n ω(x)dx ,
cioè (µk , µk+1 , . . . , µk+n ). Questo determinante è nullo poiché la matrice ha due
righe uguali. 

Osservazione 28 Se denotiamo Hn la matrice dei momenti, di elementi µi+j−2 ,


i, j = 1, . . . , n, si ha µi+j−2 = hxi−1 , xj−1 i per i, j = 1, . . . , n. Le sottomatrici
principali di testa di dimensione minore o uguale a n di una matrice dei momenti
sono matrici di Hankel. Cioè i loro elementi dipendono dalla somma degli indici
e quindi sono costanti lungo le anti-diagonali. Si può dimostrare che l’inversa di
una matrice di Hankel non singolare è una matrice di Bezout. In particolare, le
matrici di Hankel sono simmetriche. 

Osservazione 29 Col prodotto scalare su [0, 1] con ω(x) = 1, cioè definito da


R1
hf, gi = 0 f (x)g(x)dx, la sottomatrice principale di testa n × n della matrice
dei momenti è la matrice di Hilbert di elementi (1/(i+j−1)) per i, j = 1, . . . , n. 

13
Osservazione 30 Si ricorda che se A è una matrice reale simmetrica defi-
nita positiva allora l’applicazione (u, v) → uT Av da Rn × Rn in R è un
prodotto scalare.
Pn PSi osserva che, se uP= (ui )i=1,...,nP, nv = (vj−1
i )i=1,...,n , allora
n n
u T Hn v = i=1 u µ
j=1 i i+j−2 j v = h u
i=1 i x i−1
, v
j=1 j x i non è altro
che il prodotto scalare tra il polinomio di coefficienti u1 , . . . , un e il polinomio
di coefficienti v1 , . . . , vn . Dunque Hn è definita positiva e descrive il prodotto
scalare di tipo integrale tra polinomi in termini dei loro coefficienti.
Si osserva ancora che se Hn = LLT è la fattorizzazione di Cholesky di Hn ,
dove L è triangolare inferiore n × n, allora dalla condizione L−1 Hn (L−1 )T = I
segue che le righe di L−1 sono i coefficienti dei polinomi ortogonali normalizzati in
modo da avere norma 1 (polinomi ortonormali). Avevamo inoltre puntualizzato
nell’Osservazione 16, come conseguenza della formula di Christoffel Darboux, che
la matrice di Bezout Bn di pn (x) e pn−1 (x) si fattorizza nel prodotto γn Bn = L bT Lb
dove Lb è triangolare inferiore e le colonne di L b hanno per elementi i coefficienti
del generico polinomio ortonormale per cui vale L b = L−1 . Allora, confrontan-
do la relazione γn B = L T
b L b e H = (L ) L si deduce che Hn = γn−1 Bn−1 . 
−1 −1 T −1

Osservazione 31 Si osserva ancora che, fissato un prodotto scalare di tipo


integrale, per ogni n > 0 è definita la matrice dei momenti Hn di Hankel definita
positiva. Viceversa, fissato un intero n > 0 una matrice di Hankel definita
positiva Hn , definisce un prodotto scalare su Pn .

3.2 Formula di Rodrigues


Un’altra rappresentazione dei polinomi ortogonali si ottiene mediante la formula
di Rodrigues.

Teorema 32 Sia s(x) ∈ C n [a, b], tale che s(k) (a) = s(k) (b) = 0, per k =
0, 1, . . . , n − 1. Allora la funzione t(x) = s(n) (x)/ω(x) è ortogonale a ogni
Rb
polinomio di grado al più n − 1 col prodotto scalare hf, gi = a f (x)g(x)ω(x)dx.


Dim. Sia q(x) polinomio di grado al più n − 1. Allora q(x), s(n) (x)/ω(x) =
Rb (n) Rb
a
ω(x)q(x) s ω(x) (x)
dx = a q(x)s(n) (x)dx. Integrando per parti si ha

b
s(n) (x)
  Z
q(x), = [q(x)s(n−1) (x)]ba − q 0 (x)s(n−1) (x)dx
ω(x) a
Z b Z b
=− q 0 (x)s(n−1) (x)dx = · · · = (−1)n q (n) (x)s(x)dx = 0
a a

poiché q (n) = 0 essendo il grado di q minore di n. 

Osservazione 33 Nelle ipotesi del teorema, se s(n) (x)/ω(x) è un polinomio di


grado n allora esso coincide con pn (x), n-esimo polinomio ortogonale. 

14
[a, b] ω(x) sn (x) Nome
[−1, 1] 1 (1 − x2 )n Legendre
[−1, 1] (1 − x2 )−1/2 (1 − x2 )n−1/2 Chebyshev di prima specie
[−1, 1] (1 − x2 )1/2 (1 − x2 )n+1/2 Chebyshev di seconda specie
[0, +∞] e−x e−x xn Laguerre
2 2
[−∞, +∞] e−x e−x Hermite

Tabella 1: Tassonomia dei principali polinomi ortogonali

Ciò permette di esprimere i polinomi ortogonali mediante la Formula di


Rodrigues: se pn (x) è un polinomio di grado n tale che

βn dn
pn (x) = sn (x), n = 0, 1, . . . ,
ω(x) dxn

dove

βn ∈ R, sn (x) ∈ C n [a, b], : s(k) (k)


n (a) = sn (b) = 0, k = 0, 1, . . . , n − 1, (7)

allora p0 , . . . , pn sono polinomi ortogonali rispetto al prodotto scalare (2).


I valori dei βn sono scelti in modo da normalizzare i polinomi in modo più
conveniente a seconda dei casi.

4 Polinomi ortogonali specifici


Nella tabella 1 si riportano le definizioni dei polinomi ortogonali classici di
Legendre, Chebyshev di prima e di seconda specie, Laguerre ed Hermite.
I polinomi di Legendre e di Chebyshev sono casi particolari dei polinomi
ultrasferici, noti anche come polinomi di Gegenbauer cioè i polinomi ortogonali
su [−1, 1] rispetto al peso ω(x) = (1 − x2 )α , α > −1. Per questi polinomi è
possibile dare una espressione dei coefficienti in termini della funzione Γ di Eulero
http://en.wikipedia.org/wiki/Gamma_function
Z +∞
Γ(x) = tx−1 e−t dt, x > 0.
0

Tale funzione è estendibile a x ∈ C, x 6= 0, −1, −2, . . ., mediante l’espressione



1 Y (1 + n1 )x
Γ(x) = .
z n=1 1 + nx

Non solo, ma è possibile dare una forma esplicita alla funzione s(x) nella formula
di Rodrigues, infatti vale s(x) = (x2 − 1)n+α .

15
Osservazione 34 La Funzione Gamma di Eulero gode di molte proprietà
interessanti tra cui
Γ(x + 1) = xΓ(x), x ∈ R, x 6= 0, −1, −2, . . .
Γ(1) = 1
Γ(n + 1) = n!, se n è intero

Vale inoltre
dn xα Γ(α + 1)
= α(α − 1) · · · (α − n + 1)xα−n = xα−n
dxn Γ(α − m + 1)

Per ulteriori proprietà si veda [2]. 

s(n)
n (x)
Teorema 35 Se ω(x) = (1 − x2 )α , α > −1, sn (x) = (1 − x2 )α+n , allora ω(x)
(k) (k)
è polinomio di grado n e vale sn (1) = sn (−1) = 0, k = 0, 1, . . . , n − 1. Vale
inoltre
(n) n
Γ(α + n + 1)2
 
sn (x) X n
= (−1)j (1 − x)n−j (1 + x)j .
ω(x) j=0
j Γ(α + j + 1)Γ(α + n − j + 1)

(0)
s0 (1−x2 )α
Dim. Se n = 0 è ω(x) = (1−x2 )α = 1. Se n > 0 pongo z = α + n e ho

dk dk
k
(1 − x2 )z = k [(1 − x)z (1 + x)z ]
dx dx
k   j k−j
X k d z d
= j
(1 − x) k−j
(1 + x)z
j=0
j dx dx

D’altro canto vale


dm θ Γ(θ + 1)
m
x = xθ−m
dx Γ(θ − m + 1)
da cui
k  
dk 2 z
X k Γ(z + 1) Γ(z + 1)
k
(1 − x ) = (−1)j (1 − x)z−j (1 + x)z−k+j
dx j=0
j Γ(z − j + 1) Γ(z − k + j + 1)
k  
X k Γ(z + 1)2 (1 − x)k−j (1 + x)j
= (1 − x2 )z−k (−1)j
j=0
j Γ(z − j + 1)Γ(z − k + j + 1)

che diviso per ω(x) = (1−x2 )α è un polinomio in x. Inoltre, per k = 0, 1, . . . , n−1


(k) (k)
è z − k ≥ α + 1 > 0 per cui sn (−1) = sn (1) = 0. Infine per k = n vale
(n) s(n)
n (x)
sn (x) = (1 − x2 )z−n φ(x), dove φ(x) è un polinomio, per cui (1−x2 )α è un
polinomio 

16
Per i polinomi ultrasferici pn,α (x) vale allora la formula
1 dn
pn,α (x) = βn,α (1 − x2 )α+n
(1 − x ) dxn
2 α

Dal teorema precedente si possono ricavare le espressioni di an , hn in funzione


di Γ.
Si osservi che vale ancora pn,α (−x) = (−1)n pn,α (x) per cui pn,α ha coeffi-
cienti nulli per le potenze che hanno esponente con la stessa parità di n + 1.
Conseguentemente il coefficienti Bn nella relazione ricorrente a tre termini sono
nulli.

4.1 Polinomi di Legendre


I polinomi di Legendre si ottengono scegliendo [a, b] = [−1, 1] e ω(x) = 1. Sono
polinomi ultrasferici con α = 0, e la relazione ricorrente a tre termini è data da
2n + 1 n
pn+1 (x) = xpn (x) − pn−1 (x), n = 1, 2, . . .
n+1 n+1
p0 = 1, p1 = x.
In particolare

1 1
p0 (x) = 1, p1 (x) = x, p2 (x) = (3x2 − 1), p3 (x) = (5x3 − 3x).
2 2
La matrice tridiagonale associata è
x −1
 
 − 21 32 x −1 
− 23 35 x −1
 
 
− 43 74 x
 

 −1 
.
 .. .. .. 
 . . . 
i 2i+1
 

 − i+1 i+1 x −1 

.. .. ..
. . .
Qn
Osserviamo che il coefficiente principale di pn (x) per n ≥ 2 è an = i=2 2i+1
i+1 .
Denotando con pen (x) = pn (x)/an il polinomio ortogonale di grado n normalizzato
a essere monico, la relazione a tre termini diventa
2n + 1 an n an−1
pen+1 (x) = xe
pn (x) − pen−1 (x),
n + 1 an+1 n + 1 an+1
p0 (x) = 1, p1 (x) = x,
da cui si ottiene la relazione a tre termini per i polinomi monici
n2
pn (x) −
pen+1 (x) = xe pen−1 (x),
4n2 − 1
p0 (x) = 1, p1 (x) = x,

17
e la matrice tridiagonale associata è
x −1
 
 − 13 x −1 
4
− 15 −1

x

 
9
− 35 −1
 
 x 
.
.. .. ..

. . .
 
 
 2 

 − 4i2i −1 x −1 

.. .. ..
. . .
La sua simmetrizzazione è data dalla
q matrice tridiagonale che ha come elementi
2
sopra/sotto diagonali i valori di 4i2i −1 per i = 1, . . . , n − 1, cioè

− √13
 
x
 − √1 x − √215 
 3 
2 3
− √15 x − √35
 
 
3 4
 
 − √
35
x − √
63

.
 
 .. .. ..

 . . . 

 . .. i


 x − √4i2 −1 

.. ..
 
i . .
−√ 2 4i −1

Gli zeri del polinomio di Legendre di grado n sono gli autovalori della matrice
tridn ( √ i−1 2 , 0, √4ii2 −1 ), dove in generale si indica con tridn (ai , bi , ci ) la
4(i−1) −1
matrice tridiagonale n × n con elementi diagonali b1 , . . . , bn , elementi sotto
diagonali e sopra diagonali rispettivamente a2 , . . . , an e c1 , . . . , cn−1 .
Il seguente codice Octave calcola tali zeri come autovalori di questa matrice
per n = 50:
n = 50;
d=[1:n-1]./sqrt( (2*[1:n-1]-1).*(2*[1:n-1]+1));
T=diag(d,1)+diag(d,-1);
zeri=eig(T);
plot(zeri,zeros(n,1),’*’)
La figura 1 riporta gli zeri del polinomio di Legendre di grado 50.

4.2 Polinomi di Chebyshev di prima specie


I polinomi di Chebyshev di prima specie si ottengono scegliendo [a, b] = [−1, 1]
e ω(x) = (1 − x2 )−1/2 . Essi rientrano nella classe dei polinomi ultrasferici con
α = −1/2.
Si consideri la successione di polinomi definita da
Tn+1 (x) = 2xTn (x) − Tn−1 (x), n = 1, 2, . . .
(8)
T0 (x) = 1, T1 (x) = x.

18
1

0.8

0.6

0.4

0.2

−0.2

−0.4

−0.6

−0.8

−1
−1 −0.8 −0.6 −0.4 −0.2 0 0.2 0.4 0.6 0.8 1

Figura 1: Zeri del polinomio di Legendre di grado n = 50

Possiamo dimostrare il seguente risultato

Teorema 36 Per i polinomi Tn (x) definiti in (8) vale

Tn (cos(θ)) = cos nθ.


1
Inoltre essi sono ortogonali rispetto al peso ω(x) = (1 − x2 )− 2 sull’interval-
lo [−1, 1], quindi coincidono con i polinomi di Chebyshev di prima specie.
Precisamente si ha
hTn (x), Tm (x)i = 0, se n 6= m,
π
hTn (x), Tn (x)i = , per n > 0
2
hT0 (x), T0 (x)i = π,
R1 1
dove il prodotto scalare è definito da hp, qi := −1
p(x)q(x)(1 − x2 )− 2 dx.

Dim. Si pone x = cos θ, 0 ≤ θ ≤ π, e si dimostra per induzione su n che


Tn (cos θ) = cos nθ. Per n = 0, 1 la proprietà è banalmente verificata. In generale,
per l’ipotesi induttiva vale

Tn+1 (cos θ) = 2 cos θ cos nθ − cos(n − 1)θ.

Da cui si ottiene
Tn+1 (cos θ) =2 cos θ cos nθ − cos nθ cos θ − sin nθ sin θ
= cos θ cos nθ − sin nθ sin θ = cos(n + 1)θ.

Questo fatto permette di dimostrare che i polinomi Tn (x) sono ortogonali rispetto
al prodotto scalare sull’intervallo [−1, 1] con peso ω(x) = (1 − x2 )−1/2 . Infatti

19
mediante la sostituzione x = cos θ, si ottiene
Z 1
1
hTn (x), Tm (x)i = √ Tn (x)Tm (x)dx
−1 1 − x2
Z π Z π
1
= cos(nθ) cos(mθ) sin θdθ = cos(nθ) cos(mθ)dθ.
0 sin θ 0

Se m 6= n, m, n ≥ 0, allora l’integrale di cos(nθ) cos(mθ) vale 0 essendo


cos(nθ) cos(mθ) = 12 (cos((m + n)θ) + cos((m − n)θ)). Se invece n = m > 0,

essendo cos2 (nx)dx = x2 + sin(2nx) , si ha hTn (x), Tn (x)i = 0 cos2 (nθ)dθ = π2 .
R
4n Rπ
Se n = m = 0 il prodotto scalare fa chiaramente 0 dx = π. 

I polinomi Tn (x) definiti mediante le (8) sono detti Polinomi di Chebyshev


di prima specie. Dalla formula (8) segue che il coefficiente di grado massimo di
Tn (x), per n ≥ 1, è uguale a 1/2n−1 .
Un’altra conseguenza del fatto che Tn (cos θ) = cos(nθ) è che è possibile dare
(n)
una espressione esplicita agli zeri xk di Tn (x). Infatti vale il seguente
(n)
Teorema 37 Gli zeri xk , k = 1, . . . , n di Tn (x) sono

(n) (2k − 1)π


xk = cos , k = 1, . . . , n.
2n
Inoltre i valori di Tn (x) sull’intervallo [−1, 1] sono compresi tra −1 e 1, e
sull’intervallo [−1, 1] Tn (x) prende n + 1 volte il valore massimo e il valore
minimo.
(n)
Dim. I valori xk = cos (2k−1)π 2n , k = 1, . . . , n sono distinti e dalla relazione
(n) (n)
Tn (cos θ) = cos(nθ) segue Tn (xk ) = 0 per k = 1, . . . , n. Quindi xk sono
tutti gli zeri di Tn (x). Poiché Tn (cos θ) = cos(nθ), il polinomio Tn (x) è sempre
compreso tra −1 e 1 se x ∈ [−1, 1]. Inoltre Tn (x) prende i valori estremali ±1,
cioè Tn (cos θ) = ±1, se e solo se cos(nθ) = ±1, quindi se e solo se nθ = kπ, da
cui θ = kπ/n, k = 0, 1, . . . n. Quindi nell’intervallo [−1, 1] il polinomio Tn (x)
assume n + 1 volte i valori massimi e minimi. 

Un’altra proprietà molto intressante dei polinomi Tn (x), che li lega al pro-
blema della approssimazione di funzioni in norma infinito, è descritta dal
seguente

Teorema 38 Tra tutti i polinomi monici di grado n ≥ 1, il polinomio monico


1
2n−1 Tn (x) è quello che ha minima norma infinito su [−1, 1].

Dim. Si è già osservato che Tn (x) assume il valore massimo 1 e il valore minimo
−1 per n + 1 volte consecutive. Quindi kTn (x)k∞ = 1 e kTn (x)/2n k∞ = 1/2n−1 .
Supponiamo per assurdo che esista un polinomio pn monico di grado n tale che
kpn (x)k∞ < 1/2n−1 . Il polinomio pn (x) ha massimo minore di 1/2n−1 e minimo

20
maggiore di −1/2n−1 . Quindi il polinomio differenza q(x) = pn (x) − Tn (x)/2n−1
di grado al più n − 1 è negativo dove Tn ha massimo e positivo dove Tn ha
minimo. Ciò accade in n + 1 punti distinti e consecutivi di [−1, 1]. Quindi q(x)
ha n zeri. Il che è assurdo. 

La matrice tridiagonale associata ai polinomi di Chebyshev di prima specie è


 
x −1
 −1 2x −1 
 

 −1 2x −1 .


 −1 2x −1 

.. .. ..
. . .

Il coefficiente principale an di Tn (x) è an = 2n−1 per n ≥ 1. Definendo


T̃n (x) = Tn (x)/an , si ha la relazione ricorrente tra i polinomi di Chebyshev
normalizzati ad essere monici
1
Ten+1 (x) = xTen (x) − Ten−1 (x),
4
Te0 (x) = 1, Te1 (x) = x,

e la matrice tridiagonale simmetrica di cui Ten (x) è polinomio caratteristico è


 √ 
√0 2
 2 0 1 
 
1  1 0 1 

2
 .. .. .. 
 . . . 

 1 0 1
1 0
.
Osservazione 39 Vediamo che forma assume la proprietà di ortogonalità di-
screta (5) per i polinomi di Chebyshev di prima specie. Per quanto riguarda le
costanti hk si ha
Z 1
1
hk = hTk (x), Tk (x)i = Tk (x)2 √ dx.
−1 1 − x2
Con la sostituzione di variabile x = cos θ si ottiene quindi
Z π  π
se k 6= 0,
hk = cos2 kθdθ = 2
0 π se k = 0.

Per i polinomi di Chebyshev Ti (x), i = 0, 1, . . . , n − 1, la poprietà (5) di


ortogonalità discreta scritta con n − 1 al posto di n come
n
X Ti (xk ) Tj (xk ) 2
√ p σk = δi,j , i, j = 0, . . . , n − 1
k=1
hi hj

21
(n)
dove xi = xi = cos (2i−1)π
2n , per i = 1, . . . , n, sono gli zeri di Tn (x), assume una
forma più semplice. Infatti per i coefficienti σk si può dimostrare che
π
σk2 = , k = 1, . . . , n.
n
Per cui la (5) diventa
n
( n
X 2 δi,j se i 6= 0
Ti (xk )Tj (xk ) =
k=1
nδi,j se i = 0

per i, j = 0, . . . , n − 1.
La matrice C = (ci,j ) di elementi ci,j = Tj (xi−1 ) definisce la trasformata
discreta dei coseni di secondo tipo http://en.wikipedia.org/wiki/Discrete_
cosine_transform e gode della proprietà che C T C = n diag(1, 1/2, . . . , 1/2).
Inoltre il prodotto scalare nella forma discreta assume la forma
n
πX
hp(x), q(x)i = p(xk )q(xk ). (9)
n
k=1

Osservazione 40 Un polinomio q(x) di grado minore di n può essere rappre-


Pn−1
sentato nella base di Chebyshev come q(x) = k=0 ak Tk (x). Data l’ortogonalità
dei polinomi Tk (x), i valori di ak possono essere calcolati come

ak = hq(x), Tk (x)i/hTk (x), Tk (x)i.

Poiché il prodotto scalare può essere scritto in forma discreta 9, si ha


n
( 1 Pn−1
π 1 X n s=0 q(xs )Tk (xs ) per k = 0
ak = Tk (xs )q(xs ) = Pn−1
n hk s=1 2
n s=0 q(xs )Tk (xs ) per k > 0

La somma infinita

X
f (x) = ak Tk (x)
k=0

se convergente, definisce una funzione f (x) ed è chiamata serie di Chebyshev di


f (x). Data una funzione f (x) le condizioni a cui deve soddisfare f (x) affinché
esista convergente la serie di Chebyshev sono analoghe a quelle valide per la
convergenza della serie di Fourier.

La function Octave riportata nel listing 1 crea una matrice t di dimensioni


m × n che ha nella colonna j-esima i valori di Tj−1 (x) calcolati in m punti
equispaziati tra −1 e 1. Mentre la function riportata nel listing 2 crea una
matrice t di dimensioni n × n che ha nella colonna j-esima i valori di Tj−1 (x)
calcolati negli n zeri del polinomio di Chebyshev Tn (x) di prima specie di grado
n.
In questo modo, con i seguenti comandi

22
t = cheby1(100,6);
x = t(:,2);
plot(x,t(:,1),x,t(:,2),x,t(:,3),x,t(:,4),x,t(:,5),x,t(:,6))
o, più semplicemente plot(x,t), si tracciano i grafici dei primi 6 polinomi di
Chebyshev di prima specie riportati nella figura 2. Mentre scrivendo
t=chebyshev1(6);
t’*t
si ottiene la matrice numericamente diagonale
ans =

6.00000 0.00000 0.00000 0.00000 -0.00000 0.00000


0.00000 3.00000 0.00000 -0.00000 0.00000 -0.00000
0.00000 0.00000 3.00000 0.00000 -0.00000 0.00000
0.00000 -0.00000 0.00000 3.00000 0.00000 -0.00000
-0.00000 0.00000 -0.00000 0.00000 3.00000 0.00000
0.00000 -0.00000 0.00000 -0.00000 0.00000 3.00000
che conferma l’ortogonalità delle colonne di t per n = 6. Per avere invece l’orto-
gonalità delle righe, per la formula di Christoffel-Darboux occorre normalizzare
i polinomi, e quindi le colonne di t. Sapendo che la norma al quadrato della
prima colonna è h0 = π mentre le norme al quadrato delle rimanenti colonne
sono hk = π/2 per k = 1, . . . , n − 1 per verificare l’ortogonalità con Octave o
Matlab basta scrivere

T=t*sqrt(2/pi);
T(:,1)=T(:,1)/sqrt(2);
T*T’

che fornisce come risultato la matrice numericamente diagonale


ans =

1.9099 -0.0000 0.0000 -0.0000 0.0000 -0.0000


-0.0000 1.9099 0.0000 0.0000 -0.0000 -0.0000
0.0000 0.0000 1.9099 -0.0000 0.0000 0.0000
-0.0000 0.0000 -0.0000 1.9099 -0.0000 -0.0000
0.0000 -0.0000 0.0000 -0.0000 1.9099 0.0000
-0.0000 -0.0000 0.0000 -0.0000 0.0000 1.9099
in cui l’elemento diagonale coincide con n/π.

4.3 Polinomi di Chebyshev di seconda specie


I polinomi di Chebyshev di seconda specie sono definiti scegliendo [a, b] = [−1, 1]
e ω(x) = (1−x)1/2 . Rientrano nella classe dei polinomi ultrasferici, dove α = 1/2,

23
Listing 1: Function cheby1
function t=cheby1(m,n)
% function t=cheby1(m,n)
% output: t e’ la matrice mxn la cui colonna j-esima contiene i valori
% del polinomio di Chebyshev di prima specie grado j-1 campionato nei
% punti x_i=(-1+ i*2/(m-1)) per i=0,1,...,m-1
t = ones(m,n);
x = [-1:2/(m-1):1]’;
t(:,2) = x;
for j=3:n
t(:,j) = 2*x.*t(:,j-1) - t(:,j-2);
end

0.5

-0.5

-1
-1 -0.5 0 0.5 1

Figura 2: Polynomi di Chebyshev di prima specie

24
Listing 2: Function chebyshev1
function t=chebyshev1(n)
% function t=chebyshev1(n)
% output: t e’ la matrice nxn la cui colonna j-esima contiene i valori
% del polinomio di Chebyshev di prima specie grado j-1 campionato negli
% zeri del polinomio di Chebyshev di grado n: x_k=cos((2k-1)pi/(2n))
% per k=1,...,n
t = ones(n);
x = cos([pi/(2*n): pi/n : (2*n-1)*pi/(2*n)]’);
t(:,2) = x;
for j=3:n
t(:,j) = 2*x.*t(:,j-1) - t(:,j-2);
end

e soddisfano la relazione ricorrente


Un+1 (x) = 2xUn (x) − Un−1 ,
U0 (x) = 1, U1 (x) = 2x.
Infatti, procedendo come nel caso dei polinomi di Chebyshev di prima specie,
si dimostra che ponendo x = cos θ dalla relazione a tre termini segue
sin((n + 1)θ)
Un (cos θ) =
sin θ
(n) kπ
Quindi gli zeri di Un (x) sono xk = cos n+1 , k = 1, . . . , n. Inoltre dall’espressione
di Un (cos θ) si dimostra in modo analogo l’ortogonalità di questi polinomi rispetto
1
al prodotto scalare con peso (1 − x2 ) 2 .
Si osserva inoltre che
dTn dTn (cos θ) dθ d cos(nθ) d cos θ sin(nθ)
= · = / =n
dx dθ dx dθ dθ sin θ
che implica
Tn0 (x) = nUn−1 (x).
Dalla relazione a tre termini segue che la matrice tridiagonale associata ai
polinomi di Chebyshev di seconda specie è
 
2x −1
 −1 2x −1 
 

 −1 2x −1 .


 −1 2x −1 

.. .. ..
. . .
R1 √
Per le quantità hk = hUk , Uk i = −1 Uk (x)2 1 − x2 dx vale
Z π Z π
sin2 ((k + 1)θ) p
hk = 2
1 − cos θ sin θdθ = sin2 ((k + 1)θ)dθ.
0 sin2 θ 0

25
Listing 3: Function cheby2
function t=cheby2(m,n)
% function t=cheby1(m,n)
% output: t e’ la matrice mxn la cui colonna j-esima contiene i valori
% del polinomio di Chebyshev di seconda specie grado j-1 campionato nei
% punti x_i=(-1+ i*2/(m-1)) per i=0,1,...,m-1
t = ones(m,n);
x = [-1:2/(m-1):1]’;
t(:,2) = 2*x;
for j=3:n
t(:,j) = 2*x.*t(:,j-1) - t(:,j-2);
end

Da cui hk = π2 .
La function Octave riportata nel listing 3 crea una matrice u di dimensioni
m × n che ha nella colonna j-esima i valori di Uj−1 (x) calcolati in m punti
equispaziati tra −1 e 1. Mentre la function riportata nel listing 4 crea una
matrice u di dimensioni n × n che ha nella colonna j-esima i valori di Uj−1 (x)
calcolati negli n zeri del polinomio di Chebyshev Un (x) di seconda specie di
grado n.
In questo modo, con i seguenti comandi
u = cheby2(100,6);
x = [-1:2/99:1];
plot(x,u(:,1),x,u(:,2),x,u(:,3),x,u(:,4),x,u(:,5),x,u(:,6))
o più semplicemente con plot(x,u), si tracciano i grafici dei primi 6 polinomi
di Chebyshev di seconda specie riportati nella figura 3. Mentre scrivendo
u = chebyshev2(6);
u*u’
si ottiene la matrice numericamente diagonale
ans =

18.5918 -0.0000 0.0000 -0.0000 0.0000 -0.0000


-0.0000 5.7259 -0.0000 0.0000 0.0000 -0.0000
0.0000 -0.0000 3.6823 0.0000 -0.0000 0.0000
-0.0000 0.0000 0.0000 3.6823 -0.0000 -0.0000
0.0000 0.0000 -0.0000 -0.0000 5.7259 0.0000
-0.0000 -0.0000 0.0000 -0.0000 0.0000 18.5918

26
6

-2

-4

-6
-1 -0.5 0 0.5 1

Figura 3: Polinomi di Chebyshev di seconda specie

Listing 4: Function chebyshev2


function t=chebyshev2(n)
% function t=chebyshev2(n)
% output: t e’ la matrice nxn la cui colonna j-esima contiene i valori
% del polinomio di Chebyshev di seconda specie grado j-1 campionato negli
% zeri del polinomio di Chebyshev di grado n: x_k=cos(k*pi/(n+1))
% per k=1,...,n
t = ones(n);
x = cos([pi/(n+1): pi/(n+1) : n*pi/(n+1)]’);
t(:,2) = 2*x;
for j=3:n
t(:,j) = 2*x.*t(:,j-1) - t(:,j-2);
end

27
4.4 Polinomi di Laguerre
Peso ω(x) = e−x , intervallo [0, +∞]. Coefficienti della ricorrenza a tre termini:
1
An+1 = − n+1 , Bn+1 = 2n+1 n
n+1 , Cn = n+1 .

1
Ln+1 (x) = ((2n + 1 − x)Ln (x) − nLn−1 (x))
n+1
L0 (x) = 1, L1 (x) = 1 − x

Matrice tridiagonale associata


 
1−x −1
 −1 3−x −1 
1 1 1 
−2 5−x −1

diag(1, , , , . . .) 
 
2 3 4


 −3 7 − x −1 

.. .. ..
. . .

4.5 Polinomi di Hermite


2
Peso ω(x) = e−x , intervallo [−∞, +∞]. Ricorrenza a tre termini:

Hn+1 (x) = 2xHn (x) − 2nHn−1 (x)


H0 (x) = 1, H1 (x) = 2x

Matrice tridiagonale associata


 
2x −1
 −2 2x −1 
 

 −4 2x −1 


 −6 2x −1 

.. .. ..
. . .

5 Esercizi
Esercizio 1 Sia N intero positivo e ai < bi ≤ ai+1 < bi+1 , per i = 1, . . . , N − 1
numeri reali, ω(x) funzione definita su ∪N i=1 [ai , bi ] a valori positivi tale che
R bi
ai
f (x)ω(x)dx sia finito per i = 1, . . . , N e per ogni polinomio f (x). Si verifichi
PN R bi
che hp(x), q(x)i = i=1 ai p(x)q(x)ω(x)dx è un prodotto scalare, e si definiscano
pi (x), i = 0, 1, . . ., i polinomi ortogonali (normalizzati secondo un qualsiasi
criterio) rispetto a questo prodotto scalare. Dimostrare che

a) i polinomi soddisfano una relazione a tre termini;


b) gli zeri di pn (x) sono tutti reali e distinti, gli intervalli [bi , ai+1 ] contengono
al più uno zero, i rimanenti zeri appartengono all’insieme ∪N i=1 (ai , bi );

28
c) vale la formula di Christoffel-Darboux e la proprietà di ortogonalità discreta
Pn+1
k=1 pi (xk )pj (xk )wk = 0 per i 6= j dove xk sono gli zeri di pn+1 (x), e wk
delle opportune costanti positive;
d) il grado di precisione della formula di integrazione approssimata di Gauss
sull’insieme ∪N
i=1 [ai , bi ] è 2n + 1.

e) Dire se è vero che per n ≥ N ogni intervallo contiene almeno uno zero di pn .

Esercizio 2 Sia Tn = (tij ) la matrice tridiagonale n × n definita da tii = 10,


per i = 1, . . . , n, e ti,i+1 = ti+1,i = −1, per i = 1, . . . , n − 1.
a) Posto p0 = 1 e pk (x) = det(x Ik −Tk ), per k = 1, . . . , n, dove Ik è la matrice
identica di ordine k, dire se i polinomi pk (x) sono ortogonali rispetto a
qualche prodotto scalare su qualche intervallo [a, b].

b) In caso di risposta affermativa, determinare l’intervallo [a, b] e il prodotto


scalare.

Esercizio 3 Siano ϕi (x), i = 0, 1, . . . , n, polinomi ortogonali sull’intervallo


[−1, 1] rispetto al prodotto scalare
Z 1
< f, g >= ω(x)f (x)g(x) dx, dove ω(x) = (x2 − 1)2 ,
−1

e dove ϕi (x) ha grado i.

a) Si esprimano i ϕi (x) mediante la formula di Rodrigues.


b) Si calcolino i primi ϕi (x).
c) Sia V lo spazio dei polinomi di grado minore o uguale a n + 2 che si
annullano in 1 e in −1. Si descriva, mediante i ϕi (x), una base dello spazio
V costituita da polinomi ortogonali su [−1, 1] rispetto al prodotto scalare
Z 1
< f, g >= f (x)g(x) dx
−1

per lo spazio V.

Esercizio 4 Sia ϕi (x), i = 0, 1, . . . , n, un insieme di polinomi ortogonali nor-


malizzati in modo che ϕi (x) sia monico e tale che il grado di ϕi (x) sia i. Per
i = 1, 2, . . . , n si consideri la funzione razionale fi (x) = ϕi (x)/ϕi−1 (x).
a) Si dimostri che fi+1 (x) = x+Bi −Ci /fi (x), i = 1, 2, . . . , n−1 per opportune
costanti Bi e Ci .
b) Si usi questa proprietà per dimostrare che gli zeri di ϕi (x) separano quelli
di ϕi+1 (x).

29
Esercizio 5 Vogliamo estendere la teoria dei polinomi ortogonali a polinomi che
n
xi Ai , Ai ∈ Rm×m }.
P
hanno coefficienti matriciali e definiamo Pn = {P (x) =
i=0
In questo modo, P (x) è una combinazione lineare con coefficienti Ai dei polinomi
elementari xi Im . P (x) ha grado n se An 6= 0, è monico se An = Im .
Per fare questo introduciamo inoltre un “prodotto scalare” a valori matriciali
mediante l’applicazione h·, ·i : Pn × Pn → Rm×m definita da
Z b
hP, Qi = P (x)W (x)Q(x)T dx,
a
m×m
dove W (x) : [a, b] → R è tale che W (x) è continua, simmetrica e definita
positiva per ogni x ∈ [a, b], dove l’integrale di una matrice di elementi ai,j (x) è
Rb
la matrice i cui elementi sono a ai,j (x)dx.
Si dimostri preliminarmente che per ogni P, Q, R ∈ Pn e A ∈ Rm×m vale
a) hP, Qi = hQ, P iT ;
b) V = hP, P i è matrice simmetrica semidefinita positiva, inoltre V è definita
positiva se det P (x) 6≡ 0, infine V = 0 se e solo se P = 0;
c) hAP, Qi = AhP, Qi, hP, AQi = hP, QiAT , hxP, Qi = hP, xQi;
d) hP + R, Qi = hP, Qi + hR, Qi, hP, Q + Ri = hP, Qi + hP, Ri.
Definiamo P0 , P1 , . . . , Pn ∈ Pn una sequenza di polinomi ortogonali se Pi ha
grado i, è monico, e vale hPi , Pj i = 0 per ogni i 6= j. Si dimostri che
e) Se esiste una sequenza di polinomi ortogonali Pj , j = 0, . . . , n allora hPj , Pj i
è definita positiva;
f ) in Pn esiste ed è unica una sequenza Pi , i = 0, . . . , n di polinomi ortogonali
n
P
e ogni altro polinomio Q ∈ Pn può essere scritto come Ai Pi per Ai ∈
i=0
Rm×m ;
g) Pi è ortogonale ad ogni polinomio di grado minore di i;
h) i polinomi Pi (x) soddisfano una relazione a tre termini del tipo
Pj+1 (x) = (x Im − Bj )Pj (x) − Cj Pj−1 (x), j = 0, 1, . . . , n − 1
Rb R −1
b
P0 = I, P1 = x Im − a xW (x) dx a
W (x) dx

con Bj , Cj ∈ Rm×m .
 
Im
 x Im 
i) Pk (x) = xk Im − [µk , . . . µ2k−1 ]A−1 ,
 
k−1  ..
 . 
xk−1 Im
b
dove i momenti µk ∈ Rm×m sono definiti da µk = a xk W (x)dx ed Ak−1 è
R

la matrice a blocchi in cui il blocco di posto (i, j) è µi+j−2 , i, j = 1, . . . , k.

30
j) Vale un analogo della formula di Christoffel-Darboux?

Esercizio 6 Si vuole estendere il concetto


Ph Pkdi famiglia di polinomi ortogonali a
polinomi di due variabili p(x, y) = i=0 j=0 ai,j xi y j con coefficienti reali ai,j .
Per questo si definisce grado di p(x, y) il valore max{i + j : ai,j 6= 0} e si indica
con Πn l’insieme dei polinomi in due variabili di grado al più n e con Π l’insieme
di tutti i polinomi in due variabili.
Dato un prodotto scalare h·, ·i, si definisce p(x, y) ∈ Πn polinomio ortogonale
di grado n se p(x, y) ha grado n e hp(x, y), q(x, y)i = 0 per ogni polinomio
q(x, y) ∈ Πn−1 . Si definisce poi Vn lo spazio dei polinomi ortogonali di grado n.
Infine, fissati due reali a, b tali che a < b, si introduce una funzione W (x, y)
definita e continua su (a, b) × (a, b), a valori reali positivi, tale che esiste finito
RbRb
a a
W (x, y)p(x, y)dx dy per ogni polinomio p(x, y) ∈ Π.

a) Si dimostri che l’applicazione h·, ·i : Π × Π → R definita da


Z bZ b
hp(x, y), q(x, y)i = p(x, y)q(x, y)W (x, y)dx dy
a a

è un prodotto scalare.
b) Si descriva un procedimento per generare basi ortogonali di V0 , V1 , . . . , Vn
e si dimostri che dim Vk = k + 1.
c) Si verifichi che l’unione delle basi ortogonali di Vk , k = 0, . . . , n, costituisce
una base di polinomi ortogonali di Πn , e che, diversamente dal caso dei
polinomi nella sola x, tale base non è unica (a meno di multipli scalari).
Gli spazi Vk , k = 0, . . . , n sono univocamente determinati?
(n)
d) Si verifichi che se vi per i = 1, . . . , n + 1 è una base ortogonale di Vn tale
(n) (n) (n)
che hvi , vi i è indipendente da i, allora u(n) = (ui )i=1,...,n+1 := Qv (n)
(n)
è tale che ui , i = 1, . . . , n + 1 è base ortogonale di Vn per ogni matrice
ortogonale Q di dimensione n + 1.
e) Si dimostri che se p(x, y) ∈ Vn allora i polinomi xp(x, y) e yp(x, y) sono
ortogonali a tutti i polinomi di grado al più n − 2 e a tutti i polinomi
ortogonali di grado almeno n + 2.
f) Si deduca che vale un analogo della relazione ricorrente a tre termini nel
senso che per ogni p(x, y) ∈ Vn , sia xp(x, y) che yp(x, y) si scrivono come
combinazioni lineare di tre polinomi in Vn−1 , Vn e Vn+1 .
(k)
g) Denotando con v (k) un vettore di k + 1 componenti tale che vi , i =
1, . . . , k + 1 è una base di Vk , si dimostri che esistono e sono uniche matrici
An , (n + 1) × (n + 2), Bn (n + 1) × (n + 1) e Cn (n + 1) × n tali che

xv (n) = An v (n+1) + Bn v (n) + Cn v (n−1)

con v (−1) = 0, C−1 = 0. Similmente vale una proprietà analoga per yv (n) .

31
h) Si dimostri che se ϕn (x) sono polinomi ortogonali rispetto al prodotto
Rb
scalare < f (x), g(x) >= a f (x)g(x)ω(x)dx dato dal peso ω(x) : (a.b) →
R+ , allora qm,n (x, y) = ϕm (x)ϕn (y) è un insieme di polinomi in due
variabili ortogonali rispetto al peso W (x, y) = ω(x)ω(y) e qn,k−n (x, y) per
k = 0, . . . , n costituisce una base ortogonale di Vn .

Esercizio 7 Siano p0 (x), . . . , pn+1 (x) polinomi ortogonali sull’intervallo [a, b]


Rb
rispetto al prodotto scalare hp, qi = a p(x)q(x)w(x)dx, con w(x) funzione peso
Pj
positiva su [a, b], e pj (x) = i=0 pi,j xi . Siano x0 , . . . , xn gli zeri di pn+1 (x).
Si definiscano le matrici (n + 1) × (n + 1), dove gli indici scorrono da 0 a n:
A = (ai,j ), V = (vi,j ), S = (si,j ) tali che ai,j = pj (xi ), vi,j = xji , si,j = pi,j per
i ≤ j, si,j = 0 altrove.
a) Si verifichi che A = V S.
b) Si dimostri che esistono matrici diagonali D1 e D2 con elementi diagonali
positivi tali che Q = D1 AD2 è ortogonale.
c) Si dimostri che Q è il fattore ortogonale della fattorizzazione QR della
matrice D1 V , cioè D1 V = QR dove R = D2−1 S −1 . Si ricavi un algoritmo
per il calcolo dei coefficienti di pi (x), i = 0, . . . , n.
Pn
Esercizio 8 Sia Pn lo spazio dei polinomi a(x) = i=0 ai xi di grado al più n
con un prodotto scalare h·, ·i. Sia pi (x), i = 0, . . . , n una successione di polinomi
ortogonali tali che degpi (x) = i.

a) Si dimostri che la successione soddisfa una relazione a tre termini del tipo
pi+1 (x) = (ai x + bi )pi (x) − ci pi−1 (x), ci > 0, se e solo se il prodotto scalare
è tale che hxp(x), q(x)i = hp(x), xq(x)i per ogni coppia di polinomi in Pn
di grado al più n − 1.
b) Si dimostri che per ogni prodotto scalare h·, ·i su Pn esiste una matrice sim-
metrica definita positiva H di dimensione n+1 tale che hp(x), q(x)i = pT Hq
dove p e q sono i vettori (n + 1)-dimensionali dei coefficienti dei polinomi
p(x) e q(x) nella base dei monomi. Inoltre, data una matrice simmetrica
definita positiva H di dimensione n + 1 si dimostri che l’applicazione che
associa alla coppia (p(x), q(x)) il numero reale pT Hq è un prodotto scalare.
c) Si dica come è fatta la matrice nel caso in cui i polinomi ortogonali relativi
al prodotto scalare h·, ·i soddisfano una relazione ricorrente a tre termini.
Siano ξi , i = 0, . . . , n numeri reali distinti. Si verifichi che hp(x), q(x)i =
d) P
n
i=0 p(ξi )q(ξi ) è un prodotto scalare e che i polinomi ortogonali relativi a
tale prodotto verificano una relazione a tre termini.

Esercizio 9 Definiamo polinomio di Laurent di grado n l’espressione


n
X
ai (xi + x−i ), ai ∈ R, an 6= 0.
i=0

32
Se an = 1 diciamo che il polinomio di Laurent è monico, Denotiamo inoltre L
lo spazio vettoriale su R dei polinomi di Laurent. Dati a, b ∈ R tali che 1 <
a < b, definiamo un prodotto scalare su L mediante l’espressione hp(x), q(x)i =
Rb
a
p(x)q(x)w(x)dx dove w(x) è una funzione positiva e integrabile su [a, b].
Infine definiamo l’insieme {pn (x) ∈ L, n = 0, 1, 2, . . .} dei polinomi di Laurent
ortogonali tali che il grado di pn (x) è n, pn (x) è monico e hpn (x), pm (x)i = 0
per n 6= m.
Dimostrare che l’insieme dei polinomi di Laurent ortogonali non è vuoto e
poi esaminare quali delle proprietà dei polinomi ortogonali classici rimangono
valide, e in quale forma, per i polinomi ortogonali di Laurent. In particolare,
esaminare le proprietà degli zeri, la relazione ricorrente a tre termini, la proprietà
di ortogonalità discreta.
Cosa si può dire nel caso in cui 0 < a < 1 < b?

Esercizio 10 Si supponga che esista un prodotto scalare h·, ·i su P tale che

hx2 f (x), g(x)i = hf (x), x2 g(x)i, ∀f, g ∈ P (10)

e non necessariamente sia hxf (x), g(x)i = hf (x), xg(x)i.


a) Si dimostri che i polinomi ortogonali pi (x), i = 0, 1, . . . , tali che deg(pi (x)) =
i, ottenuti con questo prodotto scalare verificano una relazione di ricorrenza
del tipo pi+1 (x) = (ai x2 + bi )pi−1 (x) + ci pi (x) + di pi−2 (x) + ei pi−3 (x), per
opportune costanti ai , bi , ci , di .
b) Si dica se la matrice dei momenti, intesa come Hn = (hi,j ), hi,j =
hxi , xj i, i, j = 0, 1, . . . , n, mantiene ancora la struttura di Hankel. Si
verifichi che permutando righe e colonne di Hn con la permutazione
(1, 3, 5, 7, . . . , 2, 4, 6, 8, . . .) si ottiene una matrice 2 × 2 a blocchi formata
da 4 blocchi di Hankel e che ogni matrice di questo tipo, definita positiva,
determina un prodotto scalare su Pn che gode della proprietà (1). Si dia
un esempio di prodotto scalare su Pn , definito tramite la matrice Hn , con
la proprietà (1).
c) Sia Hn (x) la matrice ottenuta sostituendo l’ultima riga di Hn con [1, x, x2 ,
. . . , xn ]. Dire, motivando adeguatamente la risposta, se pn (x) = det Hn (x).
Pn i 2 2
d) Si decomponga p(x) = i=0 pi x come p(x) = p+ (x ) + xp− (x ) dove
p+ (x2 ) = (p(x) + p(−x))/2 e p− (x2 ) = (p(x) − p(−x))/(2x) sono la parte
pari e la parte dispari di p(x). Si consideri il prodotto scalare su [a, b] dato
Rb
da hp(x), q(x)i = a w(x)p(x)q(x)dx, associato al peso w(x) e si definisca

hp(x), q(x)i0 := hp+ (x), q+ (x)i + hp− (x), q− (x)i

Si verifichi che h·, ·i0 è un prodotto scalare tale che hx2 f (x), g(x)i =
hf (x), x2 g(x)i ed esistono polinomi p(x), q(x) tali che hxp(x), q(x)i 6=
hp(x), xq(x)i.

33
e) Si possono esprimere i polinomi ortogonali di cui al punto a) come determi-
nanti di opportune matrici a banda? Come è fatta la matrice dei momenti
del prodotto scalare del punto c)? Si analizzino le proprietà dei polinomi
pn (x) ottenuti col prodotto scalare dato nel punto c).

Esercizio 11 Siano p0 (x), . . . , pn (x), . . . ∈ P polinomi ortogonali rispetto al


Rb
prodotto scalare hp(x), q(x)i = a p(x)q(x)ω(x)dx, dove a < b e ω(x) è una
funzione peso. Dimostrare che le seguenti condizioni sono equivalenti
1. I sottospazi di polinomi P + = {q(x2 ), q(x) ∈ P} e P − {xq(x2 ), q(x) ∈ P}
sono ortogonali.
2. Vale p1 (x) = A1 x, pi+1 (x) = xAi+1 pi (x) − Ci pi−1 (x), per i = 1, 2, . . .,
dove Ai , Ci 6= 0.

3. Esistono polinomi ϕi (x) e ψi (x) di grado i tali che p2i (x) = ϕi (x2 ),
p2i+1 (x) = xψi (x2 ).
Sotto le condizioni dei punti precedenti dimostrare che ab < 0 e che i polinomi
ϕi (x) sono ortogonali rispetto ad un opportuno prodotto scalare sull’intervallo
[â, b̂] con peso ω̂. Si determinino â, b̂, ω̂. Dimostrare analoga proprietà per ψi (x).
Mettere in relazione i coefficienti della relazione a tre termini dei polinomi
ϕi (x), ψi (x) con quelli dei polinomi pi (x).
Dire come è fatta la matrice dei momenti per il prodotto scalare per cui
valgono le proprietà 1,2,3.
È possibile che con il prodotto scalare che rende ortogonali i polinomi ϕi (x)
i sottospazi P + e P − siano ortogonali? È possibile che ciò valga per il prodotto
scalare che rende ortogonali i ψi (x)?
Dire se la condizione
4. b = −a e ω(x) = ω(−x) per ogni x ∈ [a, b].
è equivalente alle condizioni 1-3.

Esercizio 12 Sia s(x) un polinomio di grado m e si consideri l’insieme V =


{p(x) = u(s(x)) : u(z) ∈ R[z]} ⊂ R[x] dove R[x] denota l’insieme di tutti i
polinomi in x a coefficienti reali. Fissati numeri reali a < b e una funzione continua
e positiva w(x) su [a, b], si consideri il prodotto scalare su V hp(x), q(x)i =
Rb
a
p(x)q(x)w(x)dx.
a) Dimostrare che V è spazio vettoriale e che in V esistono polinomi ortogonali
pk (x) tali che il grado di pk (x) è mk per k = 0, 1, 2, . . ., e hpk (x), ph (x)i = 0 per
h 6= k.
b) Dire quali proprietà dei polinomi ortogonali classici valgono e in quale forma
per i polinomi definiti su V, in particolare la relazione a tre termini, le proprietà
degli zeri e la formula di Christoffel-Darboux.

Esercizio 13 a) Dimostrare che per i polinomi di Chebyshev Tn (x) di prima


specie, posto x = 12 (z + z −1 ), con |z| = 1, vale la proprietà Tn (x) = 12 (z n + z −n ).

34
b) Dimostrare che Tm (Tn (x)) = Tmn (x), che vale la seguente generalizzazione
della ricorrenza a tre termini

Tm+n (x) = 2Tn (x)Tm (x) − Tm−n (x), m≥n

e che vale la seguente generalizzazione della formula di Christoffel Darboux


n
X
Tmn+m (x)Tmn (y)−Tmn+m (y)Tmn (x) = (Tm (x)−Tm (y))(1+2 Tmi (x)Tmi (y)).
i=1

c) Dedurre le seguenti proprietà

1. Il polinomio 1 + T2n ha n zeri di molteplicità 2 che coincidono con gli zeri


di Tn .
2. Gli zeri di Tn sono anche zeri di T(2k+1)n .
3. Se p e q sono due interi con uguale parità e p ≥ q allora l’insieme degli zeri
di Tp + Tq è l’unione degli insiemi degli zeri di T(p+q)/2 e di T(p−q)/2 .
d) Individuare un algoritmo per il calcolo di y = T2k (x) dato x, col più basso
numero di operazioni possibile.

Esercizio 14 I polinomi di Chebyshev Tn (x) e Un (x) di grado n, rispettivamente


di prima e di seconda specie, hanno ricche proprietà computazionali.
a) Dimostrare le seguenti identità

Tn+1 = xUn − Un−1


Um+n = Um Un − Um−1 Un−1
Tn+m = Tm Un − Tm−1 Un−1
(n)
b) Dimostrare
 che se v = [Un−1 (x), Un (x)]T allora v (n) = Av (n−1) , do-
0 1
ve A = . La proprietà vale anche per v (n) = [Tn−1 (x), Tn (x)]T ?
−1 2x
 
−Un−1 Un
Dimostrare che An+1 = .
−Un Un+1
c) Descrivere un algoritmo per il calcolo del valore di Un (x), Un−1 (x), Un−2 (x)
che impiega O(log n) operazioni aritmetiche. In particolare se n = 2k allora il
numero di operazioni non supera il valore di 7k. Usando invece gli autovalori e
gli autovettori di A dimostrare che il calcolo di di Un (x), Un−1 (x), Un−2 (x) per
n = 2k può essere fatto in k + γ operazioni dove γ è una costante che include
anche una estrazione di radice.
d) (Facoltativo) Ricavare una proprietà analoga a quella del punto b) per
una generica famiglia di polinomi ortogonali pn (x). Dedurre un algoritmo per
il calcolo di pn (x) in O(log n) passi paralleli avendo a disposizione n processori
aritmetici.

35
Esercizio 15 Sia ω(x) : (a, b) → R funzione continua a valori positivi tale che
Rb Rb
a
ω(x)xk dx è finito per ogni k ≥ 0 intero; sia hf, gi = a f (x)g(x)ω(x)dx il
prodotto scalare associato a ω(x). Siano pi (x), i = 0, 1, . . . , n + 1 polinomi
ortonormali rispetto a questo prodotto scalare con coefficiente principale ai > 0.
Siano infine x1 < x2 <P · · · < xn+1 gli zeri di pn+1 (x).
n
a) Dimostrare che i=0 pi (x)2 = aan+1 n
(p0n+1 (x)pn (x) − p0n (x)pn+1 (x)) e che
0 0
pn+1 (x)pn (x) − pn+1 (x)pn (x) > 0.
b) Dedurre dal punto a) che pn (x) e pn+1 (x) non possono avere zeri in comune
e che l’intervallo [xi , xi+1 ] contiene un solo zero di pn (x) per i = 1, . . . , n.
c) Dimostrare che per ogni c ∈ R il polinomio pn+1 (x) − cpn (x) ha n + 1
zeri reali distinti. Se c > 0 (risp. c < 0) essi giacciono in (a, b) ad eccezione
del maggiore (risp. minore) che sta in [a, b] solo se c ≤ pn+1 (b)/pn (b) (risp.
c ≥ pn+1 (a)/pn (a)).
d) Siano λ1 , . . . , λn i pesi delle formule di integrazione Gaussiane relative ai
nodi x1 , . . . , xn , zeri di pn (x) e al peso ω(x). Sia Ω(x) una primitiva di ω(x).
Dimostrare che esistono y0 = a < y1 < y2 < · · · < yn−1 < yn = b tali che
λi = Ω(yi ) − Ω(yi−1 ).

Esercizio 16 Siano Ln (x), n = 0, 1, 2, . . ., polinomi definiti dalla ricorrenza a


tre termini 
 L0 (x) = 2
L1 (x) = x
Ln+1 (x) = xLn (x) + Ln−1 (x)

a) Determinare esplicitamente una matrice tridiagonale Rn tale che Ln (x) =


det(xI − Rn ).
b) Dire se esiste un intervallo [a, b] e un peso ω(x) ≥ 0 su [a, b] tale che i
polinomi Ln (x) sono ortogonali rispetto al prodotto scalare definito da hf, gi =
Rb
a
f (x)g(x)ω(x)dx. Nel caso determinare a, b e ω(x).
c) Determinare esplicitamente una matrice tridiagonale Sn tale che Tn (x) =
γ det(2xI − Sn ), per una opportuna costante γ, dove Tn (x) è il polinomio di
Chebyshev di prima specie di grado n. Mettere in relazione le matrici Rn e Sn ,
e dimostrare che Ln (x) = 2i−n Tn ( 12 ix), dove i2 = −1.
d) Determinare esplicitamente gli zeri di Ln (x).
e) Dimostrare che Lm+n (x) = Lm (x)Ln (x) − (−1)n Lm−n (x) per m ≥ n.

f) Dimostrare che Ln (x) = a+ (x)n + a− (x)n , dove a± (x) = 12 (x ± x2 + 4).

Esercizio 17 Siano pn (x), n = 0, 1, . . . un insieme di polinomi ortogonali su un


intervallo rispetto ad un prodotto scalare integrale.
a) Dimostrare che esistono matrici 2 × 2
 
An x + Bn −Cn
Fn = , n = 1, 2, . . .
1 0

36
tali che per il vettore vn = [pn (x), pn−1 (x)]T vale

vn+1 = Fn+1 vn , n = 1, 2, . . . .
h i
2x −1
Dimostrare che, posto F = 1 0
, per i polinomi di Chebyshev di prima
specie Tn (x) e di seconda specie Un (x) vale
         
Tn+1 (x) n x Un+1 (x) n+1 1 n+2 0
=F , =F = −F
Tn (x) 1 Un (x) 0 1
√ √
b) Posto λ1 (x) = x + x2 − 1, λ2 (x) = x − x2 − 1, dimostrare che Un (x) =
√1 (λ1 (x)n+1 − λ2 (x)n+1 ), Tn (x) = 12 (λ1 (x)n + λ2 (x)n ). Ricavare da queste
2 x2 −1
espressioni un metodo a basso costo computazionale per il calcolo del valore di
Tn (x) e di Un (x) in un punto assegnato x ∈ [−1, 1] e valutarne il costo.
c) Dati due parametri a ∈ R \ 0, b ∈ R, si consideri la classe S dei polinomi
definita da s0 = 1, s1 = ax + b, sn+1 = 2xsn − sn−1 , per n ≥ 1. Dimostrare che
1. sn+1 (x) = (ax + b)Un (x) − Un−1 (x);
2. sn+1 (x) = µ(x)λ1 (x)n + ν(x)λ2 (x)n per opportuni µ(x) e ν(x), e indivi-
duare un algoritmo per il calcolo di s(x) in un punto x assegnato;
3. dire se esiste un intervallo [α, β] tale che per ogni valore di a e b il valore di
|sn (x)| per x ∈ [α, β] è limitato superiormente da una costante indipendente
da n.
d) Sia qn (x) definito da qn+1 (x) = (Ax + B)qn (x) − qn−1 (x), q0 (x) = 1,
q1 (x) = ax + b, dove A 6= 0 e B sono valori reali. Dire se esistono costanti d,
e tali che qn (dx + e) ∈ S. Dire se esiste un intervallo [a, b] su cui |qn (x)| sia
limitato superiormente da una costante per ogni n.

6 Risoluzione di alcuni esercizi


Risoluzione dell’esercizio 1.
Si tratta di ripetere le dimostrazioni viste a lezione adattandole al caso in esa-
me. Osserviamo che l’espressione definita di hp, qi soddisfa le proprietà del
prodotto scalare essendo bilineare, simmetrica e tale che hp, pi = 0 se e solo se
PN R bi 2
i=1 ai p (x)ω(x)dx = 0. Essendo quest’ultima una somma di quantità non ne-
gative può essere nulla se e solo se ciascuna di esse è nulla e quindi se e solo se p(x)
è nullo su tutti gli intervalli [ai , bi ]. Questo accade se e solo se p(x) è identicamente
nullo. Inoltre per la definizione data vale hxp(x), q(x)i = hp(x), xq(x)i. I polino-
mi p0 (x), p1 (x) . . . , pn (x), che si ottengono ortogonalizzando 1, x, x2 , . . . , xn col
procedimento di Gram-Schmidt, sono ortogonali e tali che grado(pi (x)) = i.
a) La proprietà del prodotto scalare hxp(x), q(x)i = hp(x), xq(x)i è sufficiente,
indipendentemente dalla natura del prodotto scalare, a dimostrare la validità
della relazione a tre termini. Infatti, i polinomi pi (x), per i = 0, . . . , n, avendo

37
grado 0, 1, . . . , n, sono linearmente indipendenti quindi costituiscono una base
di Pn . PQuindi se q(x) è un polinomio di grado m < n si può scrivere come
m
q(x) = i=0 αi pi (x) per αi ∈ R, per cui hpn , qi = 0. Questo ci permette di
dimostrare la relazione a 3 termini. Infatti essendo p0 , . . . , pn base di Pn si può
scrivere
n
X
pn+1 = βn xpn + αi pi .
i=0

Moltiplicando scalarmente per pj e usando l’ortogonalità e il fatto che hxp(x), q(x)i =


hp(x), xq(x)i segue la relazione a tre termini.
b) Gli zeri sono tutti reali come si può dedurre dal fatto che essi sono
autovalori di una matrice tridiagonale irriducibile e simmetrica. Tale matrice si
ottiene simmetrizzando la matrice che definisce la relazione a tre termini dato il
fatto che gli elementi sopra- e sotto-diagonali hanno lo stesso segno. Inoltre gli
zeri hanno molteplicità 1 poichè la matrice tridiagonale di cui questi zeri sono
autovalori ha rango almeno n − 1.
La proprietà che tutti gli zeri siano in I = ∪N i=1 (ai , bi ) è falsa in generale
come dimostra questo controesempio. Siano a1 < b1 < 0 e b2 = −a1 , a2 = −b1 ,
ω(x) = 1. In questo caso p1 (x) = x è ortogonale a p0 (x) = 1, inoltre p1 (x) si
annulla in 0 che non appartiene al dominio. In particolare, per questo prodotto
scalare, i polinomi di grado pari presentano solo potenze pari di x e quelli di
grado dispari presentano solo potenze dispari di x. Quindi tutti i polinomi di
grado dispari si annullano in 0.
In effetti la dimostrazione fatta nel caso standard per dimostrare che tutti
gli zeri stanno in (a, b) non funziona in questo caso più generale. Infatti, pur
essendo vero che il polinomio che si costruisce moltiplicando pn (x) per i fattori
di grado dispari relativi agli zeri interni al dominio non cambia segno in ciascun
intervallo, può accadere che questo segno sia diverso da intervallo ad intervallo.
Ciò non garantisce che la somma degli integrali sia non nulla. Il cambio di segno
ci può essere se nella parte compresa tra due intervalli consecutivi c’è un numero
dispari di zeri di pn (x), come accade nel controesempio mostrato.
La dimostrazione del caso standard può essere adattata per dimostrare che
nella parte compresa tra due intervalli contigui pn (x) ha al massimo uno zero
e che i rimanenti zeri stanno in I = ∪N i=1 (ai , bi ). Dimostriamo questo fatto.
Supponiamo che in qualche intervallo [bi , ai+1 ] il polinomio p(n) abbia più di
uno zero. Allora costruisco un polinomio q(x) che ha per zeri tutti gli zeri
di pn (x) negli intervalli (ai , bi ) e uno zero di pn (x) nell’intervallo [bi , ai+1 ] nel
caso in cui pn (x) dovesse avere un numero dispari, maggiore di 1, di zeri in
[bi , ai+1 ]. È evidente che se assumiamo l’esistenza di 2 o più zeri in un intervallo
[bi , ai+1 ], il polinomio q(x) ha grado minore di n. Per cui deve essere ortogonale
a pn (x). D’altro canto, per costruzione il polinomio q(x)pn (x) non cambia segno
in ciascun intervallo (ai , bi ), inoltre se q(x)pn (x) dovesse annullarsi in [bi , ai+1 ],
si annulla un numero pari di volte. Quindi il segno di q(x)pn (x) è lo stesso in
PN R b
tutti gli intervalli (ai , bi ) per cui i=1 aii q(x)pn (x)ω(x)dx non può essere nullo.
c) La dimostrazione della formula di Christoffel-Darboux si basa unicamen-
te sulla relazione ricorrente a tre termini che nel nostro caso è soddisfatta;

38
la proprietà di ortogonalità discreta si ottiene direttamente dalla formula di
Christoffel-Darboux;
d) Siano x1 , . . . , xn+1 gli zeri di pn+1 (x), per cui per la formula di ortogonalità
discreta possiamo scrivere
n+1
X
hq(x), s(x)i = q(xk )s(xk )σk
k=1

per opportuni σk e per ogni q, s polinomi di grado al più n. Allora, dato un


polinomio B(x) di grado al più 2n + 1, possiamo scrivere B(x) = pn+1 (x)q(x) +
r(x) dove q(x) è il quoziente e r(x) è il resto della divisione di B(x) per pn+1 ,
in particolare, r(x) ha grado al più n e q(x) è tale che B(xi ) = r(xi ), per i =
PN R b PN R b
1, . . . , 2n + 1. Vale allora i=1 aii B(x)ω(x)dx = i=1 aii pn+1 (x)q(x)ω(x)dx +
PN R bi
i=1 ai r(x)ω(x)dx. Il primo addendo si lascia scrivere come hpn+1 , qi, per
cui il prodotto scalare è zero essendo pn+1 ortogonale a q(x) che ha grado al
più n. Inoltre, poiché il grado di r(x) è minore di n + 1, il secondo addendo si
PN R b
può scrivere come hr, 1i per cui può essere scritto come i=1 aii r(x)ω(x)dx =
P2n+1 P2n+1
k=1 r(xk )wk = k=1 B(xk )wk . Per cui la formula integra esattamente
polinomi di grado al più 2n + 1 quindi il suo grado di precisione è 2n + 1.
b1) La proprietà 1 è vera. Infatti se per assurdo ci fosse un intervallo (ak , bk )
che non è ricoperto da zeri di polinomi ortogonali, scegliendoR f (x) continua su
I, positiva su (ak , bk ) e nulla altrove, allora approssimando I f (x)ω(x)dx con
una formula di integrazione Gaussiana, si otterrebbe valore nullo qualunque sia
il grado n, mentre l’integrale di f (x) sarebbe non nullo. Questo dà un assurdo
poiché la funzione f (x) sul compatto I può essere approssimata da un polinomio
con precisione arbitraria.
e) La proprietà è falsa. Come controesempio basta osservare che per N = 2,
e b1 = a2 , il prodotto scalare in esame coincide col prodotto scalare standard
su [a1 , b2 ]. Per cui se x1 < x2 sono gli zeri di p2 (x) basta scegliere b1 = a2 non
appartenenti a [x1 , x2 ] per avere l’assurdo.
Risoluzione dell’esercizio 2.
a) È
 
x − 10 1

 1 x − 10 1 

 . .. . .
.. .. 
x Ik − Tk = 

.

 . .
.. ..

 1 
1 x − 10

Per k = 1 è det x I1 − T1 = x − 10. Applicando la regola di Laplace all’ultima
riga si ha per k ≥ 2
  
det x Ik − Tk = (x − 10) det x Ik−1 − Tk−1 − det x Ik−2 − Tk−2 .
Quindi i polinomi pk (x) soddisfano la seguente relazione a tre termini
pk (x) = (x − 10) pk−1 (x) − pk−2 (x), con p0 = 1, p1 (x) = x − 10.

39
e si ha

p2 (x) = (x − 10)2 − 1, p3 (x) = (x − 10)3 − 2(x − 10), ...

b) Per vedere se questi polinomi sono ortogonali rispetto al prodotto scalare


Z b
< pi , pk >= ω(x) pi (x) pk (x) dx
a

per un’opportuna funzione peso ω(x) e un opportuno intervallo [a, b], si nota che
facendo il cambiamento di variabile 2 y = x − 10 e ponendo pk (2y + 10) =: uk (y),
la relazione a tre termini diventa

uk (y) = 2y uk−1 (y) − uk−2 (y), con u0 = 1, u1 (y) = 2y.

I polinomi cosı̀ definiti sono i polinomi di Chebyshev di seconda specie orto-


gonali sull’intervallo [−1, 1], cioè uk (y) = Uk (y). Si ha quindi a = 8, b = 12,
ω(x) = (1 − ((10 − x)/2)2 )1/2 e pk (x) = Uk ((x − 10)/2).

Risoluzione dell’esercizio 3.
a) I ϕi (x) sono i polinomi ultrasferici relativi alla costante α = 2. Per il
teorema 21, posto si (x) = (x2 − 1)i+2 , si ha
i  
di si (x) X i Γ(i + 3)2
= (1 − x)i−j (1 + x)j
dxi ω(x) j=0
j Γ(j + 3)Γ(i − j + 3)

i  
X i (i + 2)!2
= (1 − x)i−j (1 + x)j . (11)
j=0
j (j + 2)!(i − j + 2)!

I polinomi ultrasferici corrispondenti sono

di si (x)
ϕi (x) = βi ,
dxi ω(x)
dove le βi sono costanti. Per fissare dei valori convenienti delle βi si impone la
condizione che ϕi (1) = 1. Per x = 1 nella sommatoria (11) tutti i termini si
annullano eccetto quello per j = i, quindi
i (i + 2)!2 i

1
ϕi (1) = βi 2 = βi (i + 2)! 2i−1 , e βi = i−1 .
i (i + 2)! 2! 2 (i + 2)!
b) I primi due polinomi di calcolano direttamente

ϕ0 (x) = 1, ϕ1 (x) = x.

Per calcolare i polinomi successivi, conviene determinare prima la relazione a tre


termini, che sarà della forma

ϕi+1 (x) = xAi+1 ϕi (x) − Ci ϕi−1 (x),

40
in quanto Bi+1 = 0 per i polinomi ultrasferici. Imponendo la condizione che
ϕi (1) = 1 per ogni i si ha
Ai+1 − Ci = 1.
Inoltre è
Z 1
ai−1 hi
Ci = Ai+1 , dove hi = (−1)i ai βi i! si (x) dx.
ai hi−1 −1

Quindi Z 1
si (x) dx
i −1
Ci = −Ai+1 1 .
2(i + 2)
Z
si−1 (x) dx
−1

Dalla relazione
d
(x2 − 1)i+2 x = (2i + 4)(x2 − 1)i+1 + (2i + 5)(x2 − 1)i+2

dx
si ricava
h i1 Z 1 Z 1
2 i+2 2 i+1
(x − 1) x = (2i + 4) (x − 1) dx + (2i + 5) (x2 − 1)i+2 dx,
−1 −1 −1

cioè Z 1 Z 1
(2i + 4) si−1 (x) dx + (2i + 5) si (x) dx = 0,
−1 −1

quindi
i 2i + 4 i
Ci = Ai+1 = Ai+1 .
2(i + 2) 2i + 5 2i + 5
Allora
2i + 5 i
Ai+1 = e Ci = .
i+5 i+5
La relazione a tre termini risulta

(i + 5) ϕi+1 (x) = (2i + 5) x ϕi (x) − i ϕi−1 (x).

Quindi i primi polinomi ortogonali sono

7x2 − 1 3x3 − x
ϕ0 (x) = 1, ϕ1 (x) = x, ϕ2 (x) = , ϕ3 (x) = .
6 2
c) I polinomi di grado minore o uguale a i + 2 che si annullano in 1 e in −1
possono essere scritti nella forma

pi+2 (x) = (x2 − 1) qi (x),

41
dove qi (x) è un opportuno polinomio di grado i. Quindi il prodotto scalare
definito su V risulta
Z 1 Z 1
< pi (x), pj (x) >= pi (x) pj (x) dx = (x2 − 1)2 qi−2 (x) qj−2 (x) dx.
−1 −1

Ne segue che i ϕi (x) visti sopra costituiscono una base ortogonale per V.

Risoluzione dell’esercizio 4.
a) È ϕ0 (x) ≡ 1, quindi f1 (x) = ϕ1 (x). Dalla relazione a tre termini per i
polinomi monici ϕi (x)

ϕi+1 (x) = (x + Bi )ϕi (x) − Ci ϕi−1 (x), i ≥ 1,

in cui Ci =< ϕi (x), ϕi (x) > / < ϕi−1 (x), ϕi−1 (x) > è strettamente maggiore di
0, si ricava che
fi+1 (x) = (x + Bi ) − Ci /fi (x), i ≥ 1,
quindi le costanti richieste sono proprio quelle che valgono per la ricorrenza dei
ϕi (x).
b) Da questa relazione risulta evidente che se fi+1 (x) si annulla in xj , non
è possibile che anche fi (x) si annulli nello stesso punto. Poiché
0
fi+1 (x) = 1 + Ci fi0 (x)/fi2 (x), con f10 (x) = 1,
0
risulta che per i ≥ 1 è fi+1 (x) > 0 in ogni intervallo (αj , βj ) in cui fi+1 (x) è
definita. La fi+1 (x) ha gli stessi zeri della ϕi+1 (x), cioè ne ha esattamente i + 1.
Siano x1 , . . . , xi gli zeri di ϕi (x), che dalla teoria sappiamo essere tutti reali e
distinti. Quindi nei sottointervalli (−∞, x1 ), (x1 , x2 ), . . . , (xi−1 ), (xi , +∞) la
fi+1 (x) è crescente, quindi si annulla in ciascuno dei sottointervalli. Ne segue
che gli zeri della ϕi (x) separano gli zeri della fi+1 (x) e quindi della ϕi+1 (x).

Risoluzione dell’esercizio 5.
a) Poiché W (x) è simmetrica si ha
Z b Z b T
hP, Qi = P (x)W (x)Q(x)T dx = Q(x)W (x)P (x)T dx = hQ, P iT .
a a

b) La simmetria segue dal punto a). Sia poi v ∈ Rm con v 6= 0. Allora


Z b Z b
vT V v = v T P (x)W (x)P (x)T v dx = y(x)T W (x) y(x) dx,
a a

dove y(x) = P (x)T v. Per ogni x ∈ [a, b] la matrice W (x) è definita positiva,
quindi y(x)T W (x) y(x) ≥ 0. Ne segue che v T V v ≥ 0. Inoltre, se det P (x) 6≡ 0
b ∈ [a, b] tale che det P (b
esiste x x) 6= 0. Per continuità esiste un intorno U ⊂ [a, b]
b tale che det P (x) 6= 0 per ogni x ∈ U. Per cui y(x) 6= 0 per ogni x ∈ U
di x
e quindi y(x)T W (x)y(x) > 0 per ogni x ∈ U. Da cui v T V v > 0 essendo la
funzione integranda continua, nonnegativa su [a, b][ e positiva su tutto U.

42
Rb
Inoltre, se P (x) = O per ogni x ∈ [a, b] è V = a 0 dx = O. Viceversa, se
Rb
V = O allora a P (x)W (x)P (x)T dx = O. Se per assurdo esistesse x b tale che
x) 6= 0 allora esisterebbe un y ∈ Rm tale che P (x)T y =
P (b 6 0. Quindi, essendo
W (bx) definita positiva si avrebbe yP (b
x)W (b x)T y > 0 e per continuità questa
x)P (b
funzione rimane positiva in un intorno di x b contenuto in [a, b] quindi y T V y 6= 0
che contraddice la condizione V = 0.
c) Per A ∈ Rm×m è
Z b Z b
hAP, Qi = AP (x)W (x)Q(x)T dx = A P (x)W (x)Q(x)T dx = AhP, Qi.
a a

Le altre relazioni si dimostrano in modo analogo.


d) Seguono dalla linearità degli operatori.
e) Pj (x) è una matrice di ordine m i cui elementi sono polinomi di grado
minore o uguale ad n, e in particolare gli elementi principali hanno grado n.
Quindi det Pj (x) è un polinomio di grado m · n che si può annullare in al più
m · n punti. Per la (b) è V > 0.
f ) Si applica l’analogo del procedimento di ortogonalizzazione di Gram-
Schmidt alla sequenza di polinomi xj Im , j ≥ 0, definendo P0 (x) = Im e per
j = 1, . . . , n
i−1
X
Pi (x) = xi I + Hi,j Pj (x).
j=0

Moltiplicando scalarmente a destra per Pk (x), con k < j, si ha


i−1
X
hPi , Pk i =hxi I, Pk i + hHi,j Pj , Pk i
j=0
i−1
X
=hxi I, Pk i + Hi,j hPj , Pk i
j=0

=hxi I, Pk i + Hi,j hPk , Pk i

Imponendo la condizione di ortogonalità si ottiene Hi,j = −hxi I, Pk ihPk , Pk i−1 ,


quindi
i−1
X
Pi (x) = xi Im − hxi Im , Pk ihPk , Pk i−1 Pk (x). (12)
k=0

I polinomi ottenuti sono ortogonali per costruzione. L’ortogonalità si può


dimostrare anche induttivamente.
Dimostriamo che ogni altro polinomio di matrice
Pn Q(x) di grado al più n
si può scrivere in modo univoco come Q(x) = i=0 Ai Pi . Procediamo per
induzione su n. Se n = 0 allora Q(x)
Pn è costante e vale Q = Q ∗ I = Q ∗ P0 .
Per il passo induttivo, sia Q(x) = i=0 Qi xi e scriviamolo nella forma Q(x) =
Qn xn I + pn−1 (x) con pn−1 polinomio di matrice di grado al più n − 1. Per

43
Pn−1
l’ipotesi induttiva si può scrivere in modo univoco pn−1 (x) = i=0 Ai Pi (x) da
Pn−1
cui Q(x) = Qn xn I + i=0 Ai Pi (x). La dimostrazione è completa con An = Qn .
g) Pj (x) è ortogonale ad ogni polinomio pk (x) di grado k < j. Infatti pk (x)
può essere scritto come combinazione lineare dei polinomi ortogonali Pi (x). Poi
si sfrutta la linearità del prodotto scalare e l’ortogonalità.
h) Per la (12) è

P1 (x) = xIm − hx1 Im , P0 ihP0 , P0 i−1 P0 (x)


Z b Z b −1
= xIm − x W (x) dx W (x) dx .
a a

Per j ≥ 1 il polinomio Q(x) = Pj+1 (x) − x Pj (x) ha grado minore o uguale a j,


quindi può essere scritto come combinazione lineare
j
X
Q(x) = Hi Pi (x)
i=0

j
P
e per k ≤ j si ha hQ, Pk i = Hi hPi , Pk i = Hk hPk , Pk i. Ma
i=0

hQ, Pk i = hPj+1 , Pk i − hx Pj , Pk i,

e per k ≤ j − 2 si ha

Hk hPk , Pk i = hPj+1 , Pk i − hx Pj , Pk i = O,

quindi Hk = O per k = 0, . . . , j − 2 e resta solo

Q(x) = Hj Pj (x) + Hj−1 Pj−1 (x),

cioè  
Pj+1 (x) = x Im + Hj Pj (x) + Hj−1 Pj−1 (x). (13)

Si ha

Hj hPj , Pj i = hQ, Pj i = hPj+1 , Pj i − hx Pj , Pj i = −hx Pj , Pj i

e
Hj−1 hPj−1 , Pj−1 i = hQ, Pj−1 i = hPj+1 , Pj−1 i − hx Pj , Pj−1 i
= −hx Pj , Pj−1 i = −hPj , x Pj−1 i.
Esprimendo il polinomio x Pj−1 di grado j come combinazione lineare dei
polinomi ortogonali fino al grado j si ha hPj , x Pj−1 i = hPj , Pj i. Quindi

Hj = −hx Pj , Pj i hPj , Pj i−1 , Hj−1 = −hPj , Pj i hPj−1 , Pj−1 i−1 . (14)

44
k−1
Posto Pk = xk Im + xi Hi per k > 0, è
P
i)
i=0

k−1
X
O = hPk , P0 i = hxk Im , Im i + xi hHi , Im i
i=0

k−1
X k−1
X
= hxk Im , Im i + Hi hxi Im , Im i = µk + Hi µi ,
i=0 i=0
Rb
dove µi = hxi Im , Im i = a
xi W (x) dx. Moltiplicando per xj , con j = 1, . . . , k −1
si ha
k−1
X k−1
X
O = hxk+j Im , Im i + Hi hxi+j Im , Im i = µk+j + Hi µi+j ,
i=0 i=0

cioè
k−1
X
Hi µi+j = −µk+j , j = 0, . . . , k − 1.
i=0

In forma matriciale è
   
H0 , H1 , . . . , Hk−1 Ak−1 = − µk , µk+1 . . . , µ2k−1

dove  
µ0 µ1 ··· µk−1
 µ1 µ2 ··· µk 
Ak−1 =  ,
 
.. .. ..
 . . . 
µk−1 µk ··· µ2k−4
quindi
H0 , H1 , . . . , Hk−1 = − µk , µk+1 , . . . , µ2k−1 A−1
   
k−1

e
 
Im
k−1
X  x Im 
P k = xk Im + xi Hi = xk Im − µk , µk+1 , . . . , µ2k−1 A−1
 
.
 
k−1  ..
i=0
 . 
xk−1 Im

j) Da (13), moltiplicando a sinistra per Pj (ξ)T hPj , Pj i−1 si ha


 
Pj (ξ)T hPj , Pj i−1 Pj+1 (x) = Pj (ξ)T hPj , Pj i−1 x Im +Hj Pj (x)+Pj (ξ)T hPj , Pj i−1 Hj−1 Pj−1 (x),

e analogamente
 
Pj (x)T hPj , Pj i−1 Pj+1 (ξ) = Pj (x)T hPj , Pj i−1 ξ Im +Hj Pj (ξ)+Pj (x)T hPj , Pj i−1 Hj−1 Pj−1 (ξ).

45
Sottraendo dalla prima la seconda trasposta si ottiene

Pj (ξ)T hPj , Pj i−1 Pj+1 (x) − Pj+1 (ξ)T hPj , Pj i−1 Pj (x)

= Pj (ξ)T hPj , Pj i−1 x − ξ Pj (x) + Pj (ξ)T hPj , Pj i−1 Hj − HjT hPj , Pj i−1 Pj (x)
 

+Pj (ξ)T hPj , Pj i−1 Hj−1 Pj−1 (x) − Pj−1 (ξ)T Hj−1
T
hPj , Pj i−1 Pj (x).
Ma per la (14) è hPj , Pj i−1 Hj = HjT hPj , Pj i−1 e hPj , Pj i−1 Hj−1 = Hj−1
T
hPj , Pj i−1 =
−1
−hPj−1 , Pj−1 i , quindi resta

x − ξ Pj (ξ)T hPj , Pj i−1 Pj (x)




= Pj (ξ)T hPj , Pj i−1 Pj+1 (x) − Pj+1 (ξ)T hPj , Pj i−1 Pj (x)
+Pj (ξ)T hPj−1 , Pj−1 i−1 Pj−1 (x) − Pj−1 (ξ)T hPj−1 , Pj−1 i−1 Pj (x).

Sommando su j i termini si elidono a due a due e si ottiene


n
X
x−ξ Pj (ξ)T hPj , Pj i−1 Pj (x) = Pn (ξ)T hPn , Pn i−1 Pn+1 (x)−Pn+1 (ξ)T hPn , Pn i−1 Pn (x).
j=0

Risoluzione dell’esercizio 6.
a) Verifico che le quattro relazioni che definiscono il prodotto scalare valgono
Z bZ b
(1) simmetria hp, qi = p(x, y) q(x, y) W (x, y) dx dy
a a
Z bZ b
= q(x, y) p(x, y) W (x, y) dx dy = hq, pi
a a
Z b Z b 
(2) linearità hp1 + p2 , qi = p1 (x, y) + p2 (x, y) q(x, y) W (x, y) dx dy
a a
Z bZ b Z b Z b
= p1 (x, y) q(x, y) W (x, y) dx dy + p2 (x, y) q(x, y) W (x, y) dx dy
a a a a

= hp1 , qi + hp2 , qi
Z b Z b
(3) prodotto per scalare hα p, qi = α p(x, y) q(x, y) W (x, y) dx dy
a a
Z b Z b
=α p(x, y) q(x, y) W (x, y) dx dy = α hp, qi
a a
Z b Z b
(4) positività hp, pi = p2 (x, y) W (x, y) dx dy ≥ 0 e hp, pi = 0 se e solo
a a
se p(x, y) ≡ 0.

46
b), c) Una base di Πk è quella dei monomi xi y j−i , per i = 0, . . . , j e
j = 0, . . . , k. Quindi la dimensione di Πk è (k + 1)(k + 2)/2.
RbRb
Sia α = a a W (x, y) dx dy > 0 per ipotesi. Per semplicità suppongo che i
polinomi di Vk siano normalizzati e dimostro per induzione che l’insieme Vk è uno
spazio vettoriale di dimensione k + 1 formato, oltre che dallo 0, solo da polinomi
(0) (0)
di grado k. Per k = 0 si definisce v1 (x, y) ≡ 1/α e V0 = span (v1 ). Per k > 0
(k)
sia vi (x, y), i = 1, . . . , k + 1, una base di Vk e si considera l’insieme dei k + 2
(k+1) (k+1) (k)
polinomi wi (x, y), con wi (x, y) = x vi (x, y), per i = 1, . . . , k + 1 e
(k+1) (k)
wk+2 (x, y) = y vk+1 (x, y). A questo insieme, formato da polinomi linearmente
indipendenti di grado k + 1, si applica un procedimento di ortogonalizzazione di
(k+1)
Gram-Schmidt, ottenendo i polinomi vi (x, y), dove
(k+1) (k+1)
v1 (x, y) = w1 (x, y),

e per i = 2, . . . , k + 2
i−1
(k+1) (k+1) (k+1) (k+1) (k+1)
X
t = wi (x, y) − hwi , vj i vj (x, y), vi = t/ht, ti
j=1

Poiché una combinazione lineare di polinomi ortogonali di grado k + 1 non può


avere grado minore di k + 1, i k + 2 polinomi cosı̀ ottenuti hanno tutti grado
(j)
k + 1. Si considera poi l’unione di tutti i polinomi vi per j = 0, . . . , k + 1. Tale
insieme contiene (k + 1)(k + 2)/2, e quindi costituisce una base ortogonale di Πk .
Ne segue che i k + 2 polinomi ottenuti sopra sono sufficienti a generare Vk+1 .
La base di Vk+1 ottenuta sopra non è unica (a meno di multipli scalari).
Una base diversa potrebbe essere ottenuta applicando il procedimento di Gram-
Schmidt a partire dall’insieme dei monomi o anche semplicemente cambiando
(k+1)
l’ordine dei wi (x, y).
(n) (n) (n)
d) Si suppone per semplicità che i vj siano normalizzati, quindi hvj , vr i =
δj,r . Indicati con qi,j , i, j = 1, . . . , n + 1, gli elementi di una matrice ortogonale,

n+1
(n) (n)
X
ui = qi,j vj
j=1
e
n+1 n+1
(n) (n) (n)
X X
hui , uk i = qi,j qk,r hvj , vr(n) i = qi,j qk,j = δi,k .
j,r=1 j=1

e), f ) Sia n il grado del polinomio ortogonale p(x, y). Se q(x, y) è un


polinomio di grado minore di n − 1, si ha
Z bZ b
hx p, qi = x p(x, y) q(x, y) W (x, y) dx dy
a a
Z bZ b
= p(x, y) x q(x, y) W (x, y) dx dy = 0,
a a

47
perché x q(x, y) ha grado minore di quello di p(x, y). Se q(x, y) è un polinomio
ortogonale di grado maggiore di n + 1, si ha
Z bZ b
hx p, qi = x p(x, y) q(x, y) W (x, y) dx dy = 0
a a

perché x p(x, y) ha grado minore di quello di q(x, y) che è ortogonale. Relazioni


analoghe valgono per il polinomio y p(x, y).
Si scrive il polinomio x p(x, y) di grado n + 1 come combinazione dei polinomi
delle basi ortogonali di Vk per k = 0, . . . , n + 1
n+1
X k+1 (k) (k)
X
x p(x, y) = αk,j vj , con αk,j = hx p, vj i.
k=0 j=0

Quindi
n−1 n n+1
(n−1) (n) (n+1)
X X X
x p(x, y) = αn−1,j vj + αn,j vj + αn+1,j vj ,
j=0 j=0 j=0

dove le tre sommatorie danno polinomi in Vn−1 , Vn e Vn+1 .


(n)
g) Trascrivo la relazione precedente per il singolo polinomio vi di Vn
n−1 n n+1
(n) (i) (n−1) (i) (n) (i) (n+1)
X X X
x vi = αn−1,j vj + αn,j vj + αn+1,j vj , (15)
j=0 j=0 j=0

(i)
e considero la matrice An , il cui elemento (i, j)-esimo è αn+1,j , per i = 0, . . . , n,
(i)
j = 0, . . . , n+1, la matrice Bn , il cui elemento (i, j)-esimo è αn,j , per i = 0, . . . , n,
(i)
j = 0, . . . , n, e la matrice Cn il cui elemento (i, j)-esimo è αn−1,j , per i = 0, . . . , n,
j = 0, . . . , n − 1. In forma vettoriale si pone v (k) il vettore le cui componenti
(k)
sono vj , per j = 0, . . . , k e si ha per n ≥ 1

x v (n) = An v (n+1) + Bn v (n) + Cn v (n−1) .


Per n = 0 è
x v (0) = A0 v (1) + B0 v (0) ,
in cui v (0) = cost.
L’unicità delle matrici cosı̀ costruite dipende dal fatto che la relazione (15) esprime
(n) (k)
x vi come combinazione lineare dei vettori della base vj , j = 1, . . . , k + 1,
k = 0, . . . , n + 1. I coefficienti della combinazione lineare sono unici.
h) Suppongo per semplicità che i polinomi ϕm (x) siano normalizzati per
ogni m. I polinomi qm = ϕm (x) ϕn−m (y), m = 0, . . . n hanno grado n. Dimostro
che sono ortogonali ai polinomi di Πk per k < n rispetto al peso ω(x) ω(y). Sia
p(x, y) ∈ Πk , quindi
Xk
p(x, y) = xi pk−i (y),
i=0

48
dove pk−i (y) è un polinomio di grado al più k − i, per cui
k
X
hqm , pi = hϕm (x) ϕn−m (y), xi pk−i (y)i
i=0

k Z b
X Z b
= ϕm (x) ϕn−m (y) xi pk−i (y) ω(x) ω(y) dx dy
i=0 a a

m−1
XZ b Z b
i
= ϕm (x) x ω(x) dx ϕn−m (y) pk−i (y) ω(y) dy
i=0 a a

k Z
X b Z b
i
+ ϕm (x) x ω(x) dx ϕn−m (y) pk−i (y) ω(y) dy,
i=m a a

dove sono nulle le sommatorie il cui primo indice è maggiore del secondo. I
termini della prima sommatoria sono nulli perché ϕm (x) è ortogonale ai polinomi
xi con i < m, i termini della seconda sommatoria sono nulli perché ϕn−m (y) è
ortogonale ai polinomi pk−i (y) con i ≥ m.
Dimostro infine che i qm sono ortogonali rispetto al peso ω(x) ω(y) per
m = 0, . . . n
Z bZ b
hqm , qr i = ϕm (x) ϕn−m (y) ϕr (x) ϕn−r (y) ω(x) ω(y) dx dy
a a
Z b Z b
= ϕm (x) ϕr (x) ω(x) dx ϕn−m (y) ϕn−r (y) ω(y) dy = δm,r .
a a
Quindi qm per m = 0, . . . n costituisce una base ortogonale di Vn .

Risoluzione dell’esercizio 7.
a) A = V S segue da
j
X j
X
ai,j = pj (xi ) = pk,j xki = sk,j vi,k .
k=0 k=0

b) Si suppone che i polinomi siano monici e che quindi verifichino una


relazione a tre termini della forma
hpj , pj i
pj+1 (x) = (x + Bj+1 )pj (x) − Cj pj−1 (x), dove Cj = > 0.
hpj−1 , pj−1 i
La matrice T + xI tridiagonale di ordine n, dove T =tridiag[−Cj , Bj+1 , −1],
j = 0, . . . , n verifica la relazione
   
p0 (x) 0
 p1 (x)   .. 
(T + xI)  .

=
  . .

 ..   0 
pn (x) pn+1 (x)

49
Quindi il vettore ui = [p0 (xi ), . . . , pn (xi )]T è autovettore di T corrispondente
all’autovalore x = xi . Si indica con U la matrice la cui i-esima colonna è ui .
Quindi U = AT .
Poiché i Cj sono positivi, la matrice diagonale D, i cui elementi principali
sono di = hpj , pj i1/2 , è tale che la matrice M = D−1 T D è simmetrica. Gli
autovettori di M , normalizzati nella norma indotta dal prodotto scalare, sono
i vettori ortonormali wj = D−1 uj /γj , con γj = kD−1 uj k. Quindi la matrice
W , la cui j-esima colonna è il vettore wj , verifica la relazione W T W = I.
Si chiama Γ la matrice diagonale i cui elementi principali sono γj . Allora
W Γ = D−1 U = D−1 AT . Si pone Q = Γ−1 W T D−1 , con D1 = Γ−1 e D2 = D−1 .
c) Sostituendo A = V S si ha D1 V = QD2−1 S −1 . Poichè S è triangolare
inferiore, la matrice R = D2−1 S −1 è triangolare inferiore, quindi QR rappresenta
la fattorizzazione QR di D1 V . Questo suggerisce un algoritmo per il calcolo
dei coefficienti dei polinomi pj (x) nella base dei monomi a partire dai valori
che i pj (x) assumono negli zeri xi , i = 0, . . . , xn di pn+1 (x) e dai fattori di
normalizzazione hj = hpj , pj i:
(1) si costruisce la matrice di Vandermonde V ;
(2) con i fattori hj si costruisce la diagonale di D;
(3) si costruiscono i γj e la diagonale di D1 ;
(4) si costruisce D1 V e se ne calcola la fattorizzazione QR;
(5) la matrice S cercata è R−1 D.

Risoluzione dell’esercizio 8.
a) Per semplicità si suppongono i polinomi monici, ma ciò che segue può
essere esteso senza alterarne le conclusioni anche al caso che i polinomi non siano
monici. La successione pi (x), i = 0, . . . , n, costituisce una base dello spazio Pn e
vale hpi (x), q(x)i = 0 per ogni polinomio q(x) di grado minore di i.
Dimostriamo dapprima che se hxp(x), q(x)i = hp(x), xq(x)i per ogni coppia
di polinomi in Pn di grado al più n − 1, allora la successione soddisfa una
relazione a tre termini. Infatti pi+1 (x) per i ≥ 1 può essere espresso nella base
p0 (x), . . . , pi (x), xpi (x) nel modo seguente

pi+1 (x) = α0 p0 (x) + . . . + αi pi (x) + xpi (x)

e vale
hpj (x), pi+1 (x)i = α0 hpj (x), p0 (x)i + . . . + αi hpj (x), pi (x)i + hpj (x), xpi (x)i
= αj hpj (x), pj (x)i + hpj (x), xpi (x)i.

Se j ≤ i − 2 è hpj (x), xpi (x)i = hxpj (x), pi (x)i = 0 e hpj (x), pi+1 (x)i = 0, per
cui αj = 0. Resta

pi+1 (x) = (x + αi )pi (x) + αi−1 pi−1 (x).

50
Viceversa, dimostriamo che se la successione soddisfa una relazione a tre
termini, allora hxp(x), q(x)i = hp(x), xq(x)i per ogni coppia di polinomi in Pn
di grado al più n − 1. Per linearità basta dimostrare che la proprietà vale per i
polinomi pi (x), cioè che hxpi (x), pj (x)i = hpi (x), xpj (x)i. Se i = j la proprietà
vale per la commutatività del prodotto scalare, inoltre per la simmetria, basta
dimostrarla per i ≤ j. Quindi ciò che basta dimostrare è la relazione

hxpi (x), pj (x)i = hpi (x), xpj (x)i, i = 0, 1, . . . , j − 1. (16)

Per fare questo si osserva che per la relazione a tre termini vale

xpi (x) = pi+1 (x) − Bi+1 pi (x) + Ci pi−1 (x),


xpj (x) = pj+1 (x) − Bj+1 pj (x) + Cj pj−1 (x).

Per cui la (16) si scrive in modo equivalente come

hpi+1 (x)−Bi+1 pi (x)+Ci pi−1 (x), pj (x)i = hpi (x), pj+1 (x)−Bj+1 pj (x)+Cj pj−1 (x)i

se i ≤ j − 2, per l’ortogonalità dei polinomi la relazione diventa 0 = 0 che è


soddisfatta. Se invece i = j − 1 la relazione diventa

hpj , pj i = Cj hpj−1 , pj−1 i

che è verificata essendo Cj = hpj , pj i/hpj−1 , pj−1 i, per l’ortogonalità dei pi (x) e
per la relazione a tre termini.
b) Posto
Xn X n
p(x) = pi xi , q(x) = qi xi ,
i=0 i=0


n X
X n
hp(x), q(x)i = pi qj hxi , xj i = pT Hq, dove hi,j = hxi , xj i.
i=0 j=0

La matrice H è simmetrica per la simmetria del prodotto scalare. Inoltre è


definita positiva perché

0 < hp(x), p(x)i = pT Hp, per p 6= 0.

La dimostrazione che l’applicazione che associa alla coppia (p(x), q(x)) il numero
reale pT Hq, in cui H è una matrice simmetrica definita positiva, è un prodotto
scalare, è una verifica diretta delle proprietà di positività, di simmetria e di
bilinearità.
c) Si è visto al punto (a) che se i polinomi ortogonali verificano la relazione
a tre termini, allora hxp(x), q(x)i = hp(x), xq(x)i. Quindi per ogni i e j è

hi,j = hxi , xj i = hxi−1 , xj+1 i = hi−1,j+1 ,

per cui la H ha uguali gli elementi sulle parallele alla diagonale secondaria, cioè
è di Hankel.

51
d) Si ha
n
X n
X
p(ξi ) = pj ξij , q(ξi ) = qj ξij ,
j=0 j=0

per cui
n X
X n n
X n X
X n n
X
hp(x), q(x)i = pj ξij qk ξik = pj qk ξij ξik .
i=0 j=0 k=0 j=0 k=0 i=0

Anche in questo caso si può scrivere


n
X
hp(x), q(x)i = pT Hq, dove hj,k = ξij ξik .
i=0

Per questa matrice H valgono le stesse proprietà che per quella definita al punto
(b), in particolare che in questo modo si è definito un prodotto scalare. Inoltre
si ha
n
X n
X
hxp(x), q(x)i = ξi p(ξi )q(ξi ) = p(ξi )ξi q(ξi ) = hp(x), xq(x)i,
i=0 i=0

e questo, per quanto dimostrato al punto (a), è sufficiente per dire che i polinomi
ortogonali relativi al prodotto scalare cosı̀ definito verificano una relazione a tre
termini.

Risoluzione dell’esercizio 9
Si dà un cenno di due risoluzioni diverse.
Prima risoluzione. Si ponga t = x + x−1 e si osservi che xn+1 + x−n−1 =
(x + x−n )t − xn−1 − x−n+1 . Procedendo per induzione su n si deduca che
n

xn + x−n è un polinomio in t di grado n. Si osservi poi che essendo x reale,


risulta t = x + x−1 ≥ 2, inoltre t = t(x) è crescente per |x| > 1 e decrescente

−1 2 2
per |x| < 1. Da √ x + x = t segue x − tx + 1 = 0 per cui x = (t + t − 4)/2
2
e x = 2/(t + t − 4). Si definiscono allora pb(t) = p(x(t)). Per quanto si è
osservato queste funzioni sono polinomi di grado n in t, inoltre pb(t) è monico se
pn (x) è monico.
Inoltre, essendo a, b > 1, col cambio di variabile t = x + x−1 vale
Z b Z b
b
p(x)q(x)w(x)dx = pb(t)b
q (t)w(t)dt
b
a a
b

con b a = a√ + a−1 , bb = b + b−1 e w(t) b = w(t + t−1 )/(1 − x(t)−2 √ ) > 0 con
x(t) = (t + t − 4)/2 che è invertibile su [a, b] e vale x(t) = (t + t2 − 4)/2.
2

Segue allora che pbn (t) sono polinomi ortogonali classici e le loro proprietà si
estendono quindi a pn (x) = pbn (x + x−1 ). In particolare, poiché pbn (t) ha n zeri
(n)
reali distinti τi ∈ (b a, bb), i = 1, . . . , n, allora pn (x) ha n zeri reali distinti in (a, b)

52
 q 
(n) (n) (n)2
che sono ξi = τi + τi − 4 /2, oltre ai loro reciproci in (1/bb, 1/b
a); La
relazione a tre termini soddisfatta dai pbn (t) implica che i pn (x) soddisfano ad
una relazione del tipo

pn+1 (x) = (An+1 (x + x−1 ) + Bn+1 )pn (x) − Cn pn−1 (x),

con An+1 = 1. Vale quindi la separazione degli zeri (per la proprietà di monotonia
di t(x) nell’intervallo [a, b]), la proprietà di ortogonalità discreta, la formula di
Christoffel-Darboux, ecc.
Se 0 < a < 1 < b, la funzione t(x) = x + 1/x non è invertibile, e le proprietà dei
polinomi ortogonali classici in generale non valgono. Ad esempio, con w(x) = 1,
a = 1/2 e b = 2, vale p0 (x) = 1, p1 (x) = x + x−1 − α con α > 0. Poiché p1 (x)
ha due zeri ξ > 1 e 1/ξ, non può avere un solo zero nell’intervallo (1/2, 2).
Possiamo comunque scrivere il prodotto scalare nel modo seguente
Z b Z 1 Z b
p(x)q(x)w(x)dx = p(x)q(x)w(x)dx + p(x)q(x)w(x)dx
a a 1

x−1 avendo l’accortezza di usare


poi possiamo fare il cambio di variabile t = x + √
2
√ funzione inversa x− (t) = (t − t − 4)/2 per il primo integrale
l’espressione della
2
e x+ (t) = (t + t − 4)/2 per il secondo per cui si ottiene
Z a
b Z b
b
pb(t)b
q (t)w(t)dt
e + pb(t)b
q (t)w(t)dt
b
2 2

e = w(x− (t))/(1 − x− (t)−2 ) e w(t)


dove w(t) e = w(x+ (t))/(1 − x+ (t)−2 )
Seconda soluzione. Si procede ripetendo e adattando le dimostrazioni date nel
caso standard. I polinomi di Laurent ortogonali esistono perché sono ottenibili
in modo costruttivo mediante ortogonalizzazione di G-S. Sono linearmente
indipendenti perché di grado diverso. Per dimostrare la relazione a tre termini
si scrive
Xn
pn+1 (x) = (x + x−1 )pn (x) + αi pi (x)
i=0
−1
dato che (x + x )pn (x) è polinomio di Laurent di grado n + 1. Si moltiplica
scalarmente per pk (x), k = 0, 1, . . . , n e si ha

0 = hpk , pn+1 i = hpk , (x + x−1 )pn i + αk hpk , pk i

Inoltre vale hpk , (x + x−1 )pn i = h(x + x−1 )pk , pn i, e, poiché (x + x−1 )(xi + x−i ) =
xi+1 + x−i−1 + xi−1 + x−i+1 , il prodotto (x + x−1 )pk è un polinomio di Laurent
di grado k + 1. Allora per la proprietà di ortogonalità risulta αk = 0 per
k = 0, . . . , n − 2. Da cui la relazione a 3 termini.
Per la proprietà degli zeri si ragiona come nel caso standard. Occorre
premettere che gli zeri di pn (x) sono in coppie ξ, 1/ξ. Supponiamo per assurdo
che in (a, b) ci siano meno di n zeri di pn (x). Denotiamo con η1 , . . . , ηk quelli di

53
Qk
molteplicità dispari. Costruisco q(x) = i=1 (x − ηk )(x−1 − ηk ) che è prodotto
di polinomi di Laurent quindi è polinomio di Laurent di grado k < n. Per
l’ortogonalità deve essere hq, pn i = 0. Però sull’intervallo [a, b] il prodotto
pn (x)q(x) non cambia segno e non è identicamente nullo. Questo conduce ad un
assurdo.
In modo analogo si procede con le altre proprietà.

Risoluzione dell’esercizio 10
a) I polinomi p0 (x), p1 (x), . . . , pn−1 (x), x2 pn−2 (x) sono linearmente indipen-
denti avendo grado 0, 1, 2, . . . , n. Quindi costituiscono una base per Pn e si può
scrivere:

pn (x) = an x2 pn−2 (x) + an−1 pn−1 (x) + an−2 pn−2 (x) + · · · + a0 p0 (x).

Moltiplicando scalarmente per pk (x) per k < n si ha

0 = hpk , pn i = an hpk , x2 pn−2 i + ak hpk , pk i.

Poichè hpk , x2 pn−2 i = hx2 pk , pn−2 i e hx2 pk , pn−2 i = 0 se k + 2 < n − 2 essendo


il grado di x2 pk minore di n − 2, ne segue che ak = 0 per k < n − 4 quindi

pn (x) = an−1 pn−1 (x) + (an x2 + an−2 )pn−2 (x) + an−3 pn−3 (x) + an−4 pn−4 (x).

In particolare si ottiene
hx2 pn−2 , pn−i i
an−i = −an , i = 1, 2, 3, 4.
hpn−i , pn−i i
b) No, la struttura di Hankel non è mantenuta. Infatti se la proprietà
hxi+1 , xj i = hxi , xj+1 iP fosse verificata
Pn per ogni i, j P − 1, allora per
= 0, . . . , nP
n n n
bilinearità varrebbe h i=0 ai xi+1 , j=0 bj xj i = h i=0 ai xi , j=0 bj xj+1 i e
quindi varrebbe per ogni f (x), g(x) ∈ Pn . Deve quindi esistere (i, j) tale che
hxi+1 , xj i =
6 hxi , xj+1 i quindi hi+1,j 6= hi,j+1 e la matrice perde la struttura di
Hankel.
La condizione hx2 xi , xj i = hxi , x2 xj i implica che hi+2,j = hi,j+2 . Questa è
l’unica struttura che si conserva oltra alla simmetria. In particolare, le sotto-
matrici H1,1 = (h2i−1,2j−1 ), H2,2 = (h2i,2j ), H1,2 = (h2i−1,2j ), H2,1 = (h2i,2j−1 )
sono di Hankel. Quindi, permutando righe e colonne di H con una permutazio-
ne dispari/pari oppure pari/dispari si ottiene una matrice formata da quattro
blocchi di Hankel del tipo  
H11 H12
T .
H12 H22
Poichè se f sono i vettori dei coefficienti dei polinomi f (x) e g(x) vale
hf (x), g(x)i = f T Hg, basta considerare una matrice H con la struttura descritta
sopra che sia definita positiva e definire il prodotto scalare come sopra.
Una matrice H definita positiva con le proprietà descritte si può costruire
considerando una qualsiasi matrice derivante da un prodotto scalare di tipo

54
integrale, ad esempio la matrice di Hilbert, e per un k fissato perturbando
gli elementi hk−2i,k+2i per tutti i valori di i consentiti. Se la perturbazione è
sufficientemente piccola in valore assoluto, si ottiene ancora una matrice definita
positiva per la continuità del determinante.
Alternativamente, per n pari si può definire H1,1 = H2,2 uguali alla matrice
T
di Hilbert, H1,2 = H2,1 = 0. In questo caso vale la proprietà hx2 p(x), q(x)i =
R1 R1
hp(x), x2 q(x)i, inoltre h1, x2 i = 0 xdx mentre hx, xi = 0 1dx quindi h1, x2 i = 6
hx, xi.
c) Calcolando il determinante diPHn (x) con la regola di Laplace sull’ultima
n i n−i
riga di Hn (x) si ha det Hn (x) = i=0 x (−1) di , dove di sono i determi-
nanti delle sottomatrici ottenute cancellando l’ultima Pnriga e la colonna i + 1-
esima, per i = 0, . . . , n. Per cui hxk , det Hn (x)i = i=0 hxk , xi i det Hn (x) =
det H(x)
e dove H(x)e è la matrice ottenuta da H(x) sostituendo l’ultima riga
con [hxk , x0 i, . . . , hxk , xn i]. Ma questo è nullo per k = 1, . . . , n − 1 poiché l’ul-
tima riga coinciderebbe con la k-esima. Per cui risulta hxk , det Hn (x)i per
k = 0, 1, . . . , n − 1 e quindi det Hn (x) = pn (x).
d) h·, ·, i0 è prodotto scalare poiché è forma bilineare ed è definito positivo
essendo hp, pi0 = hp+ (x), p+ (x)i + hp− (x), p− (x)i ≥ 0 come somma di quantità
non negative, ed è nullo solo se i due addendi sono nulli, cioè solo se p(x) ≡ 0.
Vale
hx2 p(x), q(x)i0 =hx2 p+ (x2 ) + x(x2 p− (x2 )), q+ (x2 ) + xq− (x2 )i0
=hxp+ (x), q+ (x)i + hxp− (x), q− (x)i
=hp+ (x), xq+ (x)i + hp− (x), xq− (x)i
=hp(x), x2 q(x)i0
Posto p(x) = 1 e q(x) = x vale hxp(x), q(x)i0 = hx, xi0 = h1, 1i, mentre
hp(x), xq(x)i0 = h1, x2 i0 = h1, xi I due prodotti scalari sono diversi se h1, 1i = 6
h1, xi, ciò accade ad esempio con [a, b] = [0, 1] e w(x) = 1.
e) Sı̀ basta considerare la matrice A di Hessenberg inferiore pentadiagonale
con elementi sopradiagonali uguali a −1, diagonali uguali a ci , sottodiagonali
uguali a ai x2 + bi e nelle due sottodiagonali successive uguali rispettivamente a
di ed ei . Posto v = [p0 (x), . . . , pn−1 (x)]T , vale Av = pn (x)en . In particolare gli
zeri di pn (x) sono gli zeri di det A.

Risoluzione dell’esercizio 11
1 ⇒ 2 L’ortogonalità di p1 (x) = A1 x + B1 e p0 (x) = A0 implica che
hB1 , A0 i = 0 da cui B1 = 0. Induttivamente, supponiamo che Bi = 0 per
i = 0, 1, . . . , n. Dimostriamo che Bn+1 = 0. La proprietà Bi = 0 implica che per
i = 0, . . . , n vale pi (x) ∈ P + se i è pari, pi (x) ∈ P − se i è dispari (si veda anche
l’implicazione 2⇒ 3). Per cui per l’ortogonalità di P + e P − , dall’espressione
Bn+1 = −An+1 hxpn , pn i/hpn , pn i segue Bn+1 = 0.
2 ⇒ 3 Si procede induttivamente. La proprietà vale per p0 e p1 . Assumendo
valida la proprietà per pi per i = 0, 1, . . . , n si dimostra per n + 1. Dalla relazione
a tre termini
pn+1 (x) = xAn+1 pn (x) − Cn pn−1 (x)

55
se n è pari allora per ipotesi induttiva pn (x) ha monomi con potenze pari quindi
xAn+1 pn (x) ha monomi con sole potenze dispari cosı̀ come pn−1 per cui pn+1
ha monomi con sole potenze dispari. Si procede in modo analogo se n è dispari.
3 ⇒ 1 Siano r(x) ∈ P + e s(x) ∈ P − . Rappresentando r(x) e s(x) nella
base dei polinomi ortogonali si ha che r(x) è combinazione lineare dei p2i (x)
mentre s(x) è combinazione lineare dei p2i+1 (x). Dall’ortogonalità dei pi (x)
segue hr(x), s(x)i = 0.

Se fosse ab ≥ 0, ad esempio a = 0, b > 0, allora i polinomi x e 1 essendo non


negativi sull’intervallo non possono essere ortogonali.
Vale
Z b Z 0 Z b
hp2i (x), p2j (x)i = ϕi (x2 )ϕj (x2 )ω(x)dx = ϕi (x2 )ϕj (x2 )ω(x)dx+ ϕi (x2 )ϕj (x2 )ω(x)dx.
a a 0

Facendo il cambio di variabile x2 = t si ottiene


Z a2 √ Z b2 √
ω(− t) ω( t)
hp2i (x), p2j (x)i = ϕi (t)ϕj (t) √ dt + ϕi (t)ϕj (t) √ dt.
0 2 t 0 2 t

Per cui i polinomi ϕi risultano ortogonali


√ √ su [â, b̂] con 0, b̂ = max(a2 , b2 )
√ â =√
rispetto al peso ω̂(t) = I[0,a2 ] ω(− t)/(2 t) + I[0,b2 ] ω( t)/(2 t), dove I[a,b] è la
funzione indicatrice dell’intervallo [a, b]. Analogamente, per i polinomi p2i+1 (x).
Per semplicità normalizziamo i polinomi in modo che siano monici e Ai = 1.
La relazione
pi+1 (x) = xpi (x) − Ci pi−1 (x)
può essere riscritta come

xψm (x2 ) = xϕm (x2 ) − C2m xψm−1 (x2 ), i = 2m


2 2 2 2
ϕm (x ) = x ψm−1 (x ) − C2m−1 ϕm−1 (x ), i = 2m − 1

da cui
ψm (t) = ϕm (t) − C2m ψm−1 (t)
ϕm (t) = tψm−1 (t) − C2m−1 ϕm−1 (t)
Sostituendo ϕm (t) = ψm (t) + C2m ψm−1 (t) nella seconda equazione si ottiene

ψm (t) + C2m ψm−1 (t) = tψm−1 (t) − C2m−1 (ψm−1 (t) + C2m−2 ψm−2 (t)

da cui

ψm (t) = (t − C2m − C2m−1 )ψm−1 (t) − C2m−1 C2m−2 ψm−2 (t)

Analogamente,

ϕm+1 (t) = (t − C2m+1 − C2m )ϕm (t) − C2m C2m−1 ϕm−1 (t)

56
Essendo x2i+1 ortogonale a x2j , la matrice dei momenti H = (µi+j ) ha
elementi nulli se i + j è dispari. Per cui mediante una permutazione di righe e
colonne che porta in testa gli indici 0, 2, 4, 6, . . . e in coda gli indici 1, 3, 5, 7, . . .
la matrice si trasforma in una matrice diagonale a blocchi con blocchi diagonali
di Hankel. I due blocchi diagonali definiscono il prodotto scalare dei polinomi
ϕi (t) e ψi (t).

Non è possibile che i polinomi ϕi (t) e ψi (t) godano di analoghe proprietà.


Infatti essi sono ortogonali sull’intervallo [â, b̂], dove âb̂ = 0 mentre si è visto che
è necessario che gli estremi dell’intervallo abbiano prodotto negativo.
Un’altra motivazione è nel fatto che gli analoghi dei termini Bi nella relazione
a tre termini dei ψi (t) e ϕi (t) dovrebbero essere nulli, cioè deve essere C2m +
C2m−1 = Cm + C2m+1 = 0. Ciò è impossibile essendo Ci > 0.

Risoluzione esercizio 12 Si dà un cenno alla risoluzione senza dare i


dettagli che vengono
Pn lasciati allo studente.P
n
Se u(z) = i=0 ui z i allora u(s(x)) = i=0 ui s(x)i , per cui ogni polinomio
p(x) = u(s(x)) di V è una combinazione lineare dei polinomi s(x)k i cui coefficienti
sono i coefficienti di u(z). Per cui V è lo spazio vettoriale su R una cui base è
data dalle potenze intere di s(x). Si osserva che il grado di s(x)k è km.
Applicando il procedimento di ortogonalizzazione di Gram-Schmidt ai poli-
nomi s(x)k , k = 0, 1, 2, . . . , si ottiene un insieme di polinomi pk (x) tali che pk (x)
ha grado km e hpk (x), ph (x)i = 0 se h 6= k.
Poiché i pk (x) hanno grado diverso sono linearmente indipendenti.
Inoltre pk (x) è ortogonale a ogni polinomio q(x) ∈ V di grado più basso poiché
q(x) è combinazione lineare dei pi (x) per i = 0, . . . , k − 1 e hpi (x), pk (x)i = 0.
Dalla definizione data di prodotto scalare segue la proprietà hs(x)p(x), q(x)i =
hp(x), s(x)q(x)i. Da questa segue a sua volta una relazione ricorrente a tre termini
del tipo
pk+1 (x) = (s(x)Ak+1 − Bk+1 )pk (x) − Ck pk−1 (x).
Infatti poiché q(x) := s(x)pk (x) ∈ V e il suo grado è m(k + 1), si ha che
pk+1 − αk+1 q(x) ha grado al più mk per un opportuno αk+1 . Quindi possiamo
Pk
scrivere pk+1 (x) = αk+1 s(x)pk (x) + i=0 αi pi (x).
Moltiplicando scalarmente per pj (x), per j = 0, 1, . . . , k, usando l’ortogonalità
si ha 0 = αk+1 hs(x)pk (x), pi (x)i + αi hpi (x), pi (x)i = αk+1 hs(x)pi (x), pk (x)i +
αi hpi (x), pi (x)i. Poiché il grado di s(x)pi (x) è mi, se i < k il prodotto scalare
hs(x)pi (x), pk (x)i è nullo per cui αi = 0. Ne segue la relazione a tre termini.
Inoltre se i polinomi sono normalizzati in modo che Ak = 1 allora il coefficiente
Ck è positivo essendo Ck = Ak+1 hphs(x)p k (x),pk−1 i
k−1 (x),pk−1 (x)i
hpk+1 (x),pk+1 (x)i
, con Ak+1 = hs(x)p k (x),pk+1 (x)i
,
hpk (x),pk (x)i
per cui Ck = AAk+1
k hpk−1 (x),pk−1 (x)i
> 0.
Dalla relazione a tre termini segue la formula di Christoffel-Darboux nella
forma
k
X 1
γk (pk+1 (x)pk (y) − pk+1 (y)pk (x)) = (s(x) − s(y)) pi (x)pi (y).
h
i=0 i

57
la cui dimostrazione si ottiene trascrivendo e adattando la dimostrazione vista a
lezione.
Le proprietà degli zeri richiedono un’analisi più approfondita. Dimostriamo
ora che se s(x) è iniettiva su [a, b] allora pk (x) ha almeno k zeri semplici nell’in-
tervallo (a, b). Se gli zeri semplici in (a, b) fossero h < k, siano essi x1 , . . . , xh ,
Qh
allora il polinomio i=1 (s(x) − s(xi )) ∈ V ha grado mh < mk = grado(pk (x)),
per cui hpk , qi = 0. Però il polinomio pk (x)q(x) non cambia segno su [a, b] purchè
s(xi ) 6= s(xj ) per i 6= j che è vero se s(x) è iniettiva. Questa proprietà implica
che hpk , qi =
6 0. Assurdo.
Si può dimostrare che se s(x) è iniettivia su [a, b] allora gli zeri sono esat-
tamente k. Sotto questa ipotesi di iniettività si può dire sostanzialmente di
più. Definiamo uk (z) tale che pk (x) = uk (s(x)), per semplicità assumiamo che
s0 (x) > 0 per x ∈ (a, b) e poniamo α = s(a), β = s(b), t = s(x), x = s−1 (t), dove
α < β. Risulta
b β β
w(s−1 (t))
Z Z Z
pk (x)ph (x)w(x) = uk (t)uh (t) 0 −1 dt = uk (t)uh (t)w̃(t)dt
a α s (s (t)) α

dove w̃(t) > 0 su [α, β]. Quindi i polinomi uk (t) sono polinomi ortogonali
standard rispetto a un prodotto scalare di tipo integrale con opportuno peso.
Vale quindi
uk+1 (z) = (Ak+1 z + Bk+1 )uk (z) − Ck uk−1 (z).
Situazione analoga si ha se s0 (x) < 0 poichè α R> β e cambiando segno al peso e
α
scambiando gli estremi di integrazione si ottiene β uk (t)uh (t)w̃(t)dt dove β < α
e w̃(t) > 0, quindi ancora un prodotto scalare. Per cui i polinomi uk (y) sono
polinomi ortogonali su [min(α, β), max(α, β)] col prodotto scalare con peso w̃(y).
Quindi gli zeri di uk (t) sono reali e semplici compresi tra (min(α, β), max(α, β))
e godono dell proprietà di interlacing. Se s(x) è iniettiva su [a, b], gli zeri reali di
pk (x) in (a, b) sono esattamente k e godono della proprietà di interlacing. Inoltre
tutte le proprietà dei polinomi ortogonali valide per gli uk (y) si estendono a
pk (x) avendo cura di sostituire la variabile y con s(x). Si riottengono quindi la
relazione a 3 termini, la formula di Christoffel-Darboux ecc.
Se s(x) non è monotona l’integrale tra a e b può essere spezzato in una somma
di integrali su intervalli che hanno per estremi a, b e i punti in cui si annulla la
derivata prima di s(x). Trattando ogni singolo intervallo come sopra si deduce
che gli zeri di pk (x) sono compresi tra minx∈[a,b] s(x) e maxx∈[a,b] s(x). Non è
detto che siano k o che siano distinti. Infatti se τ è zero di uk (t) allora i valori
di ξ tali che s(ξ) = τ sono zeri di pk (x). Poiché s(x) non è iniettiva ci possono
essere al più m valori di ξ per cui s(ξ) = τ .
Un tipo di analisi similare può essere condotta come segue. Si osserva ancora
che dalla relazione a tre termini segue che per il polinomio pk (x) = uk (s(x)) vale
pk (x) = det(Tk − s(x)I) dove Tk è una matrice reale simmetrica k × k. Per cui,
avendo Tk autovalori λ1 , . . . , λk reali distinti che coincidono con gli zeri di uk (x).
Allora il polinomio pk (x) si annullerà in quei valori di x per cui s(x) = λi ,
per i = 1, . . . , k. Se s(x) è iniettivo su [a, b] allora dalla proprietà di interlacing

58
dei λi segue la proprietà di interlacing degli zeri di pk (x). Se s(x) non è iniettivo
possono esserci più soluzioni dell’equazione s(x) = λi .

Risoluzione esercizio 13 a) Essendo |z| = 1 vale z = cos θ + i sin θ per


θ ∈ [0, 2π), per cui x = cos θ. Quindi Tn (x) = cos nθ = 12 (z n + z −n ) essendo
z n = cos nθ + i sin nθ.

b) Le proprietà del punto b) si dimostrano ponendo x = cos θ e usando un po’


di trigonometria. Infatti, Tm (Tn (x)) = Tm (Tn (cos θ)) = Tm (cos nθ). Posto σ =
nθ si ha Tm (cos nθ) = Tm (cos σ) = cos mσ = cos mnθ = Tmn (cos θ) = Tmn (x).
Per la ricorrenza a tre termini, partendo dalla relazione standard

Tn+1 (x) = 2xTn (x) − Tn−1 (x)

e sostituendo x con Tm (x), dalla proprietà Tn (Tm (x)) = Tmn (x) si ottiene

Tm(n+1) (x) = 2Tm (x)Tmn (x) − T(n−1)m (x).

Una via più contosa è applicare un po’ di trigonometria:

Tn (x)Tm (x) = cos nθ cos mθ


Tm−n (x) = cos(m − n)θ = cos mθ cos nθ + sin mθ sin nθ

per cui 2Tn (x)Tm (x) − Tm−n (x) = cos nθ cos mθ − sin nθ sin mθ = cos(m + n)θ =
Tm+n (x).
Per la formula di Christoffel-Darboux generalizzata si parte dalla formula
standard
n
X 1
γn (Tn+1 (x)Tn (y) − Tn (x)Tn+1 (y)) = (x − y)( Ti (x)Ti (y)),
h
i=0 i

an 1 n−1
dove γn = an+1 hn = π essendo per i polinomi di Chebyshev an = 2 e h0 = π,
hi = π/2 per i > 0. Per cui si può scrivere
n
X
Tn+1 (x)Tn (y) − Tn (x)Tn+1 (y) = (x − y)(1 + 2 Ti (x)Ti (y)).
i=1

Ora, ponendo x = Tm (u) e y = Tm (v), poiché Tn+1 (x) = Tn+1 (Tm (u)), per il
punto a) si ottiene Tn+1 (x) = T(n+1)m (u). Analogamente si ha Tn (y) = Tmn (v)
da cui
n
X
Tnm+m (u)Tnm (v)−Tnm (u)Tnm+m (v) = (Tm (u)−Tm (v))(1+2 Tmi (u)Tmi (v)).
i=1

c1) Ponendo m = n nella relazione a tre termini generalizzata si ottiene

1 + T2n (x) = 2Tn (x)2

59
da cui la tesi.
c2) Segue dalla relazione T(2k+1)n = T2k+1 (Tn (x)) essendo T2k+1 (0) = 0
poiché il grado di T2k+1 (x) è dispari.
c3) Dalla relazione a tre termini generalizzata segue

Tm+n (x) + Tm−n (x) = 2Tm (x)Tn (x)

Ponendo p = m + n, q = m − n, allora p e q hanno stessa parità inoltre


m = (p + q)/2, n = (p − q)/2, da cui Tp (x) + Tq (x) = 2T p+q (x)T p−q (x) e quindi
2 2
la tesi.
d) Una possibilità è usare la formula a tre termini generalizzata con m = n
che fornisce
T2n (x) = 2Tn (x)2 − 1.
Questa espressione, a partire da T2 (x) = 2x2 − 1, permette di calcolare il valore
di Tn (x) per n = 2k in 3k = 3 log2 n operazioni. Una possibile function in Matlab

function y=T(k,x)
% function y=T(k,x)
% calcola y=cheby(x) di grado 2^k
y=2*x^2-1;
for i=1:k-1
y=2*y^2-1;
end
Usando invece l’idetità Tm2 (x) = Tm (Tm (x)), si ha Tn (x) = T√n (T√n (x))
per cui il calcolo di Tn (x) è ridotto a calcolare T√n (x) in due valori diversi della
variabile, x e T√n (x). Per cui se cn è il costo del calcolo di Tn (x) si ha cn = 2c√n .
k
Poiché c2 = 3, per n = 22 si ha cn = 2c22k−1 = 22 c22k−2 = · · · = 2k c2 = 3 log2 n.
Una possibile function in Matlab è
function y=T1(k,x)
% function y=T1(k,x)
% calcola y=cheby(x) di grado 2^(2^k)
if k==0
y=2*x^2-1;
return
end
y1=T1(k-1,x);
y=T1(k-1,y1);

Risoluzione esercizio 14
a) Ci sono vari modi per dimostrare queste proprietà. Ne diamo un cenno.
Soluzione 1. La relazione Tn+1 = xUn − Un−1 si può dimostrare sfruttando
l’interpretazione matriciale. Infatti sappiamo che Tn+1 è uguale al determinante
della matrice tridiagonale An+1 = (ai,j ) con a1,1 = x, ai,i = 2x per i =

60
2, . . . , n + 1, ai,i+1 = ai+1,i = −1. Mentre Un+1 è il determinante della matrice
tridiagonale Bn+1 = tridn+1 (−1, 2x, −1). Calcolando det An+1 con lo sviluppo
per righe sulla prima riga si ha det An+1 = x det Bn − det Bn−1 = xUn − Un−1 .
La relazione Um+n = Um Un −Um−1 Un−1 si ottiene calcolando il determinante
di Um+1 sviluppando lungo la riga m-esima. Analogamente per Tm+n .
Soluzione 2. Si possono dimostrare le proprietà del punto a) usando la
relazione del punto b) sulla matrice An+1 . Ad esempio, per la terza proprietà
Um+n = Um Un − Um−1 Un−1 basta infatti scrivere l’identità An+m =An Am,
Um−2 U
ricordare che le colonne di Am sono date rispettivamente da − , m−1
Um−1 Um
e leggere l’identità sulle colonne.
Soluzione 3. Si procede per induzione. Ad esempio, per dimostrare la
relazione Tn+1 = xUn − Un−1 , si verifica come passo iniziale che la relazione vale
per n = 1 e per n = 2 (questo discende dalla definizione di T2 , T3 , U0 , U1 , U2 ). Si
dimostra poi il passo induttivo, cioè supponendo valida la Tn+1 = xUn − Un−1 ,
si dimostra Tn+2 = xUn+1 − Un . Per questo si scrive Tn+2 = 2x ∗ Tn+1 − Tn
si applica l’ipotesi induttiva su Tn+1 e su Tn (è per questo che il passo iniziale
richiede la verifica per n = 1, 2) e si ha

Tn+2 = 2x(xUn − Un−1 ) − (xUn−1 − Un−2 ) = x[2xUn − Un−1 ] − [2xUn−1 − Un−2 ]

che, per la relazione ricorrente a 3-termini dei polinomi di Chebyshev coincide


con xUn+1 − Un .
Anche la relazione Um+n = Um Un − Um−1 Un−1 si può usare l’induzione nel
seguente modo. Supponendo che l’identitá vale per ogni m e n positivi tali che
m + n = k si dimostra che vale per ogni m e n positivi tali che m + n = k + 1.
Il passo iniziale si ha per k = 2 e include solo il caso di m = n = 1.

b) Se x = (x1 , x2 )T allora Ax = (x2 , 2xx2 − x1 )T per cui Av (1) = (U1 , U2 )T


e, induttivamente Av (n−1) = v (n) . La proprietà si applica nello stesso modo
anche a v (n) =[Tn−1 , Tn ]T .
−U0 U1
Vale A2 = . Cioè la prima colonna è −v (1) mentre la seconda
−U1 U2
colonna è v (2) . Per la proprietà precedente la matrice An+1 = An · A ha la prima
colonna uguale a −v (n) e la seconda uguale a v (n+1) .

c) Basta calcolare la matrice An che ha elementi Un−2, Un−1 Un . Se n = 2


k

−a b
basta calcolare k quadrati di matrici del tipo H = . Vale
−b c

a2 − b2
 
b(c − a)
H2 =
−b(c − a) c2 − b2
Basta quindi eseguire 3 quadrati, 3 addizioni e una moltiplicazione per un
totale di 7k operazioni. Portando A in forma diagonale A = SDS −1 , si ha
k k
A2 = SD2 S −1 per cui la parte di calcolo che dipende da k si √
limita a valutare
k
2
D . Gli elementi diagonali di D sono λ e 1/λ, dove λ = x ± x2 − 1, per cui

61
k
basta calcolare λ2 . Ciò richiede k moltiplicazioni. Il numero delle operazioni
restanti non dipende da k

Risoluzione esercizio 15
a) Dalla formula di Christoffel-Darboux,
n
X 1 an
pi (x)pi (y) = (pn+1 (x)pn (y) − pn+1 (y)pn (x))
i=0
(x − y) an+1

aggiungendo e togliendo al secondo membro l’espressione pn+1 (y)pn (y) si ottiene


1 an
((pn+1 (x) − pn+1 (y))pn (y) − pn+1 (y)(pn (x) − pn (y)).
(x − y) an+1

Prendendo il limite per y → x, il secondo membro diventa


an
(p0 (x)pn1 (x) − pn+1 (x)p0n (x))
an+1 n+1
Pn
da cui la tesi essendo il primo membro i=0 pi (x)2 . Una dimostrazione alterna-
tiva consiste nel procedere per induzione su n. Un’altra dimostrazione consiste
nel derivare la formula di Christoffel-Darboux rispetto a x e porre poi y = x.
Poiché si è supposto ai > 0 ne segue

p0n+1 (x)pn (x) − pn+1 (x)p0n (x) > 0, (17)

b) Dalla precedente espressione segue che pn (x) e pn+1 (x) non possono avere
zeri comuni. Infatti, se ξ fosse uno zero comune, cioè pn (ξ) = pn+1 (ξ) = 0,
allora l’espressione in (17) sarebbe nulla in ξ. Inoltre se ξ < η sono due zeri
consecutivi di pn+1 (x) allora necessariamente p0n+1 (ξ)p0n+1 (η) < 0, inoltre dalla
(1) si ha p0n+1 (ξ)pn (ξ) > 0 e p0n+1 (η)pn (η) > 0, e quindi pn (ξ) e pn (η) hanno
segno opposto. Quindi pn (x) ha un numero dispari di zeri in ogni intervallo [ξ, η].
Poiché pn (x) ha n zeri, nessun intervallo [ξ, η] può contenere più di uno zero.
c) Si considera la funzione razionale r(x) = pn+1 (x)/pn (x) che ha asintoti
verticali nei punti x1 , . . . , xn . La derivata prima di r(x) è r0 (x) = (p0n+1 (x)pn (x)−
pn+1 (x)p0n (x))/pn (x)2 che è positiva in virtù della (1). Quindi r(x) è crescente
in ogni intervallo [xi , xi+1 ] per i = 0, . . . , n con x0 := a, xn+1 := b. La tesi segue
dal fatto che gli zeri di pn+1 (x) − cpn (x) sono le ascisse dei punti di intersezione
del grafico di r(x) con la retta di equazione y = c.
d) Essendo Ω0 (x) = ω(x) > 0, la funzione Ω(x) è crescente. Poiché Ω(x)
è definita a meno di una costante additiva, può essere scelta in modo che
Pn Rb
Ω(a) = 0. Poiché λi > 0 e i=1 λi = a ω(x)dx = Ω(b) − Ω(a) = Ω(b), i valori
zi = λ1 + · · · + λi stanno nel codominio di Ω(x) per i = 1, 2, . . . , n e per la
crescenza di Ω(x) e poiché zi+1 > zi , esistono punti yi ∈ [a, b] tali che Ω(yi ) = zi
e yi < yi+1 . Vale quindi Ω(yi ) − Ω(yi−1 ) = λi .

Risoluzione esercizio 16

62
a) Vale Ln (x) = det(xI − Rn ) dove Rn è la sottomatrice principale di testa di
0 2
 
−1 0 1 
 0 −1 0 1
 
.

−1 0 1

 
 
.. .. ..
. . .

Si può procedere per induzione: per n = 1 e n = 2 la proprietà è verificata. Per il passo


induttivo è sufficiente calcolare det(xI − Rn+1 ) con la regola di Laplace sull’ultima riga.

b) I polinomi ortogonali soddisfano una relazione a tre termini del tipo Pn+1 (x) =
(An x + Bn )Pn (x) − Cn Pn−1 (x), dove assumendo An = 1 risulta Cn > 0. Nel caso dei polinomi
Ln (x) il segno di Cn è negativo. Quindi i polinomi Ln (x) non possono essere ortogonali con
nessun prodotto scalare di tipo integrale con peso ≥ 0 su [a, b].

c) Per i polinomi di Chebyshev di prima specie Tn (x) vale


x −1 2x −2
   
−1 2x −1  −1 2x −1 

.. .. ..
 1 
.. .. ..

Tn (x) = det   = det(diag( , 1, . . . , 1) 
   
 . . .  2  . . . 

 −1 2x −1  −1 2x −1
−1 2x −1 2x
1
quindi Tn (x) = 2
det(2xI − Sn ) dove

0 2
 
1 0 1 
 
Sn = 
 .. .. .. .

 . . . 
 1 0 1
1 0
Confrontando Rn e Sn si osserva che differiscono per i segni sulla sottodiagonale. Per
cui, se definiamo D = diag(1, i, i2 , . . . , in−1 ), dove i2 = −1, si ha D−1 Sn D = iRn . Quin-
di D−1 (2xI − Sn )D = 2xI − iRn = i( 2 xI − Rn ). Prendendo il determinante ne segue
i
det(2xI − Sn ) = (i)n det( 2 xI − Rn ) da cui Tn (x) = 12 (i)n det( 2 xI − Rn ) = 12 (i)n Ln ( 2 x).
i i i
quindi Ln (x) = 2i−n Tn ( 21 ix).

d) Gli zeri di Ln (x) si ottengono da quelli di Tn (x) essendo Ln (x) = 2i−n Tn ( 12 ix).

e) Dimostriamo prima che Tm+n (x) = 2Tm (x)Tn (x) − Tm−n (x). Ponendo x = cos t,
essendo Tn (cos t) = cos(nt), l’identità si riscrive come cos((m + n)t) = 2 cos(mt) cos(nt) −
cos((m − n)t) che è verificata. Applicando il punto c) a questa identità dei polinomi di
Chebyshev si ottiene
1 1 1 1
Ln+m (x) =2i−n−m Tm+n ( ix) = 2i−n−m (2Tm ( ix)Tn ( ix) − Tm−n ( ix))
2 2 2 2
−n−m 1 m+n 1 m−n
=2i ( i Ln (x)Lm (x) − i Lm−n (x))
2 2
−2n
=Lm (x)Ln (x) − i Lm−n (x) = Lm (x)Ln (x) − (−1)n Lm−n (x)

f) Si cercano soluzioni speciali della ricorrenza a tre termini del tipo pn = λn . Imponendo
la relazione a tre termini si ottiene λn+1 = xλn + λn−1 da cui λ2 − xλ − 1 = 0 che dà

λ = 12 (x ± x2 + 4). Una generica combinazione lineare delle due soluzioni speciali, cioè
√ √
sn = α 12 (x + x2 + 4)n + β 21 (x − x2 + 4)n verifica ancora la ricorrenza a tre termini.

63
Scegliendo α = β = 1 si ha che sono verificate anche le due condizioni iniziali s0 = 2, s1 = x
per cui sn = Ln .

Risoluzione esercizio 17
a) Essendo pn (x) polinomi ortogonali su un intervallo rispetto ad un prodotto
scalare integrale essi verificano la relazione a tre termini

pn+1 (x) = (A bn+1 )pn (x) − C


bn+1 x + B bn pn−1 (x).

Scrivendo per esteso la relazione da dimostrare


    
pn+1 An+1 x + Bn+1 −Cn+1 pn
=
pn 1 0 pn−1
si ottiene in modo equivalente
pn+1 (x) = (An+1 x + Bn+1 )pn (x) − Cn+1 pn−1 (x),
pn (x) = pn (x),

che è chiaramente soddisfatta per An+1 = A en+1 , Bn+1 = A


en+1 , Cn+1 = C
en .
Per i polinomi
 Tn (x) e Un (x) vale An = 2, Bn = 0, Cn = −1 per cui
2x −1
Fn = =: F . Quindi
1 0
       
Tn+1 Tn T x
=F = ··· = Fn 1 = Fn .
Tn Tn−1 T0 1
Similmente si ottiene
       
Un+1 n 2x n 1 n+1 1
=F =F F =F .
Un 1 0 0
       
0 1 n+1 1 n+2 0
Inoltre, poiché F =− si ha F = −F .
1 0 0 1
b) È sufficiente portare F in forma diagonale. Infatti, dal polinomio
√ carat-
teristico di F , λ2 − 2xλ + 1 ricaviamo gli autovalori λ1(x) = x+ x2 − 1 e
√ λ λ2
λ2 (x) = x − x2 − 1 e la matrice degli autovettori V = 1 per cui vale
1 1
la fattorizzazione F = V diag(λ1 , λ2 )V −1 . Essendo
 
−1 1 1 −λ2
V =
λ1 − λ2 −1 λ1
si ottiene   n   
λ λ2 λ1 0 1 1 −λ2
F = 1
n
1 1 0 λn2 λ1 − λ2 −1 λ1
che assieme al punto a) implica
1 1 n
Un (x) = √ (λn+1 − λn+1 ), Tn (x) = (λ + λn2 ).
2 x2 − 1
1 2
2 1

64
Un modo diverso per dimostrare le relazioni per x ∈ [−1, 1] consiste nel porre
x = cos θ per cui λ1 = cos θ + i sin θ, λ2 = λ̄1 . In questo modo la relazione
Tn (x) = 12 (λ1 (x)n + λ2 (x)n ) diventa Tn (cos θ) = cos(nθ). Analogamente per
Un (x).
Un altro modo consiste nel procedere per induzione su n.
Un algoritmo per il calcolo di Tn (x) consiste nei seguenti passi

1. λ1 = x + i 1 − x2
2. s1 = λn1
3. s2 = 1/s1
4. Tn (x) = (s1 + s2 )/2
Il costo computazionale è dato da 3 operazioni aritmetiche e il calcolo di una
radice quadrata per il punto 1; una esponenziazione al passo 2 che può essere
calcolata mediante quadrati successivi avendo decomposto n in base 2, per
un costo di O(log n) operazioni. Oppure mediante logaritmo ed esponenziale
attraverso il calcolo di due funzioni elementari. I punti 3 e 4 richiedono il calcolo
di un numero finito di operazioni aritmetiche.
Analogamente si procede per Un (x).
c1) Dalla analisi del punto a) segue
         
sn+1 (x) n s1 (x) n ax + b n 1 n 0
=F =F = (ax + b)F +F
sn (x) s0 (x) 1 0 1
da cui sn+1 (x) = (ax + b)Un (x) − Un−1 (x).
Un’altra possibilità consiste nel procedere per induzione su n.
c2) Dalla forma diagonale di F n e dal punto precedente si ottiene inoltre
   
sn+1 (x) n n −1 ax + b
= V diag(λ1 (x) , λ2 (x) )V
sn (x) 1

da cui sn (x) = 12 (λn1 + λn2 ) + 2√x12 −1 ((a − 1)x + b)(λn1 − λn2 ).


Lo stesso risultato si poteva ottenere dalla analoga rappresentazione di Un e
Un−1 essendo sn (x) = (ax + b)Un (x) − Un−1 (x).
Per un algoritmo di calcolo, procedendo come al punto b) si ottiene un costo
O(log2 n).
c3) Segue dal punto c1 essendo Un (x) limitato uniformemente in modulo da
una costante su un intervallo [α, β] con −1 < α < β < 1.
Si osserva, che dalle relazioni ottenute al punto c2) segue che per x 6∈ [−1, 1],
il valore di sn (x) cresce esponenzialmente con n.
d) Ponendo sn (x) = qn (dx + e) si hanno le relazioni
qn+1 (dx + e) = (A(dx + e) + B)qn (dx + e) − qn−1 (dx + e)
qn+1 (dx + e) = 2xqn (dx + e) − qn−1 (dx + e)
da cui Ad = 2, Ae + B = 0, quindi d = −2/A, e = −B/A.

65
Riferimenti bibliografici
[1] D. Bini, M. Capovani, O. Menchi, Metodi Numerici per l’Algebra Lineare.
Zanichelli, Bologna, 1987.
[2] R. Bevilacqua, D. Bini, M. Capovani e O. Menchi, Metodi numerici,
Zanichelli, Bologna 1992.
[3] J. Stoer, R. Burlisch, Introduction to Numerical Analysis, Third Edition,
Springer, 2002.

66

Potrebbero piacerti anche